FRCOphth Part 1: 400 SBAs and CRQs H Nikki Hall, Robert G Peden
INDEX
×
Chapter Notes

Save Clear


Exam Paper 1: Structured Practice PaperChapter 1

 
Questions: SBAs
120 SBAs to be answered in 3 hours
For each question, please select the single best answer.
 
Anatomy
  1. In the superior orbital fissure which nerves pass outside the common tendinous ring?
    1. Lacrimal nerve, frontal nerve, abducens nerve
    2. Lacrimal nerve, frontal nerve, trochlear nerve
    3. Lacrimal nerve, nasociliary nerve, abducens nerve
    4. Lacrimal nerve, nasociliary nerve, frontal nerve
  2. What is the horizontal diameter of the adult cornea (in mm)?
    1. 9.2
    2. 10.6
    3. 11.2
    4. 11.7
  3. Regarding the osteology of the skull, which of the following statements is false?
    1. Layers of compact bone are separated by a layer of spongy bone known as diploë
    2. Sutural ligaments are connective tissue between bones
    3. Sutures are mobile articulations
    4. The bones of the orbit include bones of the cranium and facial bones
  4. Where does the frontal sinus drain into the nose?
    1. Inferior meatus
    2. Middle meatus2
    3. Sphenoethmoidal recess
    4. Superior meatus
  5. Which bones make up the medial wall of the orbit?
    1. Ethmoid, lacrimal, maxilla
    2. Ethmoid, nasal, lacrimal, maxilla
    3. Sphenoid, ethmoid, lacrimal, maxilla
    4. Sphenoid, nasal, palatine
  6. Regarding the Meibomian glands, which of the following statements is correct?
    1. Anterior blepharitis is characterised by Meibomian gland dysfunction
    2. The Meibomian glands lie posterior to the grey line
    3. There are approximately 150 Meibomian glands in the upper lid and 75 in the lower
    4. They produce a mucinous secretion which is part of the tear film
  7. Which of the following correctly describes the anatomy of the lacrimal gland?
    1. Parasympathetic fibres reach the lacrimal gland via the pterygopalatine ganglion and zygomaticotemporal nerve
    2. The lacrimal gland lies in the lacrimal fossa, formed by the lacrimal bone and maxilla
    3. The lacrimal gland receives parasympathetic and sensory fibres, and has no sympathetic innervation
    4. The parasympathetic innervation of the lacrimal gland is derived from the lacrimatory nucleus of the trigeminal nerve
  8. Which one of the following statements accurately describes a property of extraocular muscle, when compared to skeletal muscle elsewhere in the body?
    1. Muscle fibres of larger diameter are more peripherally placed in the muscle
    2. The epimysium is generally thicker
    3. The muscle fibres are more loosely packed, separated by connective tissue
    4. The muscle spindles are shorter (<10 μm) and less numerous
  9. Which one of the following structures does not pass through the foramen spinosum?
    1. Meningeal branch of the facial nerve
    2. Meningeal branch of the mandibular nerve
    3. Middle meningeal artery
    4. Middle meningeal vein
  10. Which of the following nerves is not a branch of the facial nerve?
    1. Buccal nerve
    2. Frontal nerve3
    3. Mandibular nerve
    4. Zygomatic nerve
  11. Which of the following intracranial bleeds is typically a result of bleeding from the meningeal arteries?
    1. Extradural haematoma
    2. Intracerebral haemorrhage
    3. Subarachnoid haemorrhage
    4. Subdural haematoma
  12. Schwalbe's line represents the termination of which corneal layer?
    1. Bowman's layer
    2. Corneal endothelium
    3. Corneal stroma
    4. Descemet's membrane
  13. At which location is the lens capsule thinnest?
    1. Anterior pole
    2. Immediately anterior to the zonular insertion
    3. Immediately posterior to the zonular insertion
    4. Posterior pole
  14. What is the site of attachment of the vitreous base?
    1. Fovea
    2. Optic disc and peripapillary region
    3. Peripheral retina and pars plana
    4. Posterior lens capsule
  15. From outermost to innermost, which is the correct sequence of the layers of Bruch's membrane?
    1. Basement membrane of the choriocapillaris, outer collagenous layer, elastin layer, inner collagenous layer, basement membrane of the retinal pigment epithelium (RPE)
    2. Basement membrane of the choriocapillaris, outer collagenous layer, inner collagenous layer, elastin layer, basement membrane of the RPE
    3. Basement membrane of the RPE, outer collagenous layer, elastin layer, inner collagenous layer, basement membrane of the choriocapillaris
    4. Basement membrane of the RPE, outer collagenous layer, inner collagenous layer, elastin layer, basement membrane of the choriocapillaris
  16. What is the point of insertion of the lateral canthal tendon?
    1. Eisler's tubercle
    2. Frontozygomatic suture4
    3. Whitnall's tubercle
    4. Zygomaticomaxillary suture
  17. Which of the following statements about the innervation of the iris is true?
    1. Sensory innervation of the iris is derived from branches of the lacrimal nerve
    2. The parasympathetic nerves to the iris synapse in the ciliary ganglion
    3. The principal nerves to the dilator pupillae muscle originate in the Edinger–Westphal nucleus
    4. The sphincter pupillae muscle is innervated by the long ciliary nerves
  18. From posterior to anterior, what is the orientation of the optic canal?
    1. Inferolateral
    2. Inferomedial
    3. Superolateral
    4. Superomedial
  19. Which statement about the ciliary body structure is false?
    1. The blood supply to the ciliary body is via the anterior ciliary arteries and the long posterior ciliary arteries
    2. The ciliary body has a greater nasal anteroposterior length than temporal anteroposterior length
    3. The middle radial layer of the ciliary muscle is continuous with the corneoscleral meshwork
    4. The outer longitudinal layer of the ciliary muscle attaches to the scleral spur
  20. Which of the following statements about the blood supply to the iris is incorrect?
    1. Iris capillaries are non-fenestrated
    2. Most of the anastomoses between the major and minor arterial circles run through the anterior border layer
    3. The major arterial circle of the iris receives blood from the anterior ciliary arteries
    4. The minor arterial circle of the iris is at the level of the collarette
  21. Into which vein do the lymphatics from the right side of the head and neck drain?
    1. Right external jugular vein
    2. Right internal jugular vein
    3. Right lymphatic duct
    4. Right subclavian vein
  22. Regarding the microscopic structure of a blood vessel:
    1. Fragmentation of the external elastic lamina is typically seen in giant cell arteritis
    2. The internal elastic lamina is in the intima5
    3. The media is the main connective tissue layer of the blood vessel
    4. Veins have relatively thicker media than arteries
 
Biochemistry
  1. Which one of the following substances is found in a lower concentration in aqueous compared with plasma?
    1. Ascorbate
    2. Glucose
    3. Lactate
    4. Sodium
  2. Regarding collagen synthesis, which of the following statements is false?
    1. Collagen usually contains glycine at every third amino acid
    2. Glycosylation of collagen is vitamin C-dependent
    3. Post-translational modification occurs both in the endoplasmic reticulum and extracellularly
    4. The triple helix structure of collagen is linked by covalent S–S bonds
  3. Regarding matrix metalloproteinases (MMPs):
    1. Endogenous inhibitors called TIMPs are present in normal tissues
    2. MMPs are magnesium-dependent proteins
    3. They are not required for normal tissue growth
    4. They are released mainly by B cells in acute inflammation
  4. Regarding lens proteins, which of the following statements is false?
    1. Cytoplasmic interdigitations between lens fibres contain channels formed by MIP26 (aquaporin-0)
    2. Epithelial growth factor stimulates the differentiation of lens epithelial cells into lens fibres
    3. The differentiation of lens epithelial cells into lens fibres is characterised by a loss of organelles
    4. The genes encoding α-crystallin are found on chromosomes 21 and 11
  5. Which of the following statements about glycosaminoglycans is false?
    1. Glycosaminoglycans are composed of long chains of repeating disaccharides
    2. Glycosaminoglycans are relatively inflexible molecules
    3. Glycosaminoglycans are strongly hydrophilic
    4. Glycosaminoglycans have a high positive charge6
  6. Which of the following best describes the actions of the corneal endothelial cell pump?
    1. There is active transport of potassium into the aqueous
    2. There is active transport of sodium into the aqueous
    3. There is passive diffusion of sodium into the aqueous
    4. There is passive diffusion of potassium into the corneal endothelial cell
  7. Which one of the following vitamins is not lipid soluble?
    1. Vitamin A
    2. Vitamin C
    3. Vitamin D
    4. Vitamin K
  8. Which of the following statements regarding pericytes is false?
    1. Pericytes are contractile
    2. Pericytes are embedded in the basement membrane of vessels
    3. Pericyte coverage is highest in the choroid
    4. Pericytes have an important role in the blood–brain barrier
  9. Which of the following statements about vascular endothelial growth factor (VEGF) is false?
    1. Aflibercept is an anti-VEGF monoclonal antibody
    2. TNF α upregulates VEGF synthesis
    3. VEGF binds to receptors that act via intracellular tyrosine kinase
    4. VEGF enhances permeability of vessels to macromolecules
  10. Regarding photoreceptor segment renewal, which of the following statements is false?
    1. Diurnal phagocytosis of rod outer segment tips occurs at first light
    2. Melatonin synthesised by photoreceptors modulates rod disc shedding
    3. Rod outer segments complete their renewal cycle every 21 days
    4. Rod discs are phagocytosed in groups of approximately 200
 
Embryology, growth and development
  1. Which embryological cell population is the lens derived from?
    1. Mesoderm
    2. Neural crest-derived mesenchyme
    3. Neural ectoderm
    4. Surface ectoderm7
  2. In which quadrant are iris colobomas typically found?
    1. Inferonasal
    2. Inferotemporal
    3. Superonasal
    4. Superotemporal
  3. Which pharyngeal arch gives rise to the orbicularis oculi muscle?
    1. First pharyngeal arch
    2. Second pharyngeal arch
    3. Third pharyngeal arch
    4. Fourth pharyngeal arch
  4. At what gestational age does myelination of the optic nerve start?
    1. Week 10
    2. Week 16
    3. Month 5
    4. Month 7
  5. Regarding the changes in the vitreous with age, which of the following is incorrect?
    1. Hyaluronate pools and redistributes within the vitreous with age
    2. Other than water, the main constituents of adult vitreous are type II collagen and hyaluronate
    3. The vitreous is homogeneous during childhood, without visible collagen fibrils
    4. The vitreous liquefies first in the periphery causing the posterior vitreous cortex to detach
 
Genetics
  1. Which of the following conditions is not a result of autosomal trisomy?
    1. Down syndrome
    2. Edwards syndrome
    3. Klinefelter syndrome
    4. Patau syndrome
  2. Regarding transcription, which of the following statements is true?
    1. Introns are removed from the RNA strand before it leaves the nucleus
    2. RNA synthetase is the enzyme primarily responsible for creating the resultant RNA molecule
    3. The coding strand of DNA is copied to make a mirror RNA strand
    4. The resultant RNA molecule extends in a 3′ to 5′ direction8
  3. Which mode of inheritance is responsible for the transmission of Leber's hereditary optic neuropathy?
    1. Autosomal dominant
    2. Autosomal recessive
    3. Mitochondrial
    4. X-linked
  4. Regarding mosaicism and chimerism, which of the following statements is false?
    1. All women are mosaics with respect to the X-chromosome genes
    2. Chimeras have genetically distinct cell populations arising from one zygote
    3. Individuals with cancer exhibit somatic mosaicism
    4. Infliximab is a murine–human chimeric monoclonal antibody
  5. Regarding genetic linkage, which of the following statements is correct?
    1. Families undergoing linkage analysis need not express the phenotype of the allele in question
    2. Genetic loci on the same chromosome are normally expected to segregate together
    3. The aim of linkage analysis is to discover how often two loci are separated by mitotic recombination
    4. The probability of crossing over between two loci on the same chromosome is proportional to the distance between them
  6. Which of the following statements regarding histones is false?
    1. Histone octamers are found at each nucleosome
    2. Histones can be dynamically modified to influence gene expression
    3. Histones contain a large quantity of lysine and arginine
    4. Histones have a high negative charge
  7. Which of the following statements about mitochondrial genetics is false?
    1. DNA in mitochondria is double-stranded
    2. DNA polymerase β is the principal enzyme in mitochondrial DNA replication
    3. Mitochondrial chromosomes are circular
    4. Mitochondrial genomes contain genes for tRNA and rRNA
  8. A 28-year-old patient with Reis–Buckler corneal dystrophy is pregnant. She already has a 10-year-old daughter who is unaffected. What is the risk of her child inheriting the condition?
    1. None
    2. 25%9
    3. 25% if her husband is a carrier
    4. 50%
  9. Regarding gene therapy, which of the following is not a vector used to transfer DNA into a cell?
    1. Adenoviruses
    2. Liposomes
    3. Retroviruses
    4. Ribozymes
 
Immunology
  1. Which of the following is not a feature of the innate immune system?
    1. Complement cascade
    2. Mucosa-associated lymphoid tissue (MALT)
    3. NK cells
    4. Recognition of pathogen-associated molecular patterns (PAMPs)
  2. Which of the following is not an antigen presenting cell?
    1. B cell
    2. Dendritic cell
    3. Monocyte
    4. T cell
  3. Regarding major histocompatibility complex (MHC) class I molecules, which of the following statements is false?
    1. An example of an MHC class I molecule is HLA B27
    2. MHC class I molecules are present on every nucleated cell
    3. They present antigenic peptides on the surface of the cell
    4. Viral proteins presented with MHC class I molecules are recognised by CD4+ T cells
 
Investigations and imaging
  1. In which one of the following conditions would the electro-oculogram (EOG) be normal?
    1. Advanced Stargardt macular dystrophy
    2. Best's vitelliform macular dystrophy10
    3. Optic neuritis
    4. Total retinal detachment
  2. Which one of the following tissues appears bright on a T1-weighted MRI?
    1. Air
    2. Calcium
    3. Cerebrospinal fluid (CSF)
    4. Fat
 
Microbiology
  1. Regarding endophthalmitis, which of the following statements is false?
    1. Bacillus cereus is frequently implicated in chronic low-grade endophthalmitis
    2. Coagulase-negative staphylococci are frequently isolated in endophthalmitis following cataract surgery
    3. Exogenous endophthalmitis is more common than endogenous endophthalmitis
    4. Pseudomonas endophthalmitis causes severe tissue destruction by enzyme secretion
  2. Which of the following is used for the culture of Mycobacterium tuberculosis?
    1. Löwenstein–Jensen medium
    2. MacConkey agar
    3. Sabouraud's agar
    4. Theyer–Martin medium
  3. Which of the following antibiotics acts by inhibiting bacterial DNA-dependent RNA polymerase?
    1. Cefuroxime
    2. Chloramphenicol
    3. Rifampicin
    4. Vancomycin
  4. What is the minimum time required for dry heat sterilisation at 160°C?
    1. 30 minutes
    2. 60 minutes
    3. 120 minutes
    4. 180 minutes
  5. Which of the following statements about Acanthamoeba is true?
    1. Acanthamoeba keratitis is typically painless due to damage to corneal sensory nerve endings11
    2. Chlorhexidine 0.02% and propamidine isetionate applied topically for a week will sterilise most Acanthamoeba infections
    3. It can be grown on Escherichia coli nutrient-deficient agar
    4. It is the most common cause of infective keratitis in contact lens wearers
  6. By what mechanism does botulinum toxin A inhibit neurotransmission?
    1. Acetylcholine receptor antagonism
    2. Blockade of postsynaptic sodium channels
    3. Inhibition of acetylcholine exocytosis
    4. Intracellular blockade of acetylcholine production
  7. Which of the following viruses does not typically cause keratitis?
    1. Adenovirus
    2. Cytomegalovirus
    3. Herpes simplex virus
    4. Varicella zoster virus
  8. Which of the following is more typical of Toxocara than toxoplasma infection?
    1. Infection via ingestion of animal faeces
    2. Leukocoria
    3. Vitritis
    4. White chorioretinal lesions
  9. Regarding Haemophilus influenzae:
    1. It is a cause of orbital cellulitis
    2. It is a Gram positive anaerobe
    3. Encapsulated serotypes are less virulent
    4. Vaccination covers most virulent serotypes
 
Optics
  1. What is the magnification of a loupe of power +16 dioptres?
    1. × 4
    2. × 8
    3. × 32
    4. × 64
  2. An object lies outside the centre of curvature of a concave mirror. How is the image formed by the mirror best described?
    1. Real, inverted, diminished
    2. Real, inverted, enlarged12
    3. Virtual, erect, diminished
    4. Virtual, erect, enlarged
  3. A crown glass prism in air has an apical angle of 4°. What will the angle of deviation be in the position of minimum deviation?
    1. 16°
  4. How is the image formed by a convex lens of an object located within the first focal point best described?
    1. Erect, real, inside the second focal point, diminished
    2. Erect, virtual, further from lens than object, magnified
    3. Inverted, real, outside the second focal point, magnified
    4. Inverted, virtual, at infinity, diminished
  5. What is the maximum amplitude of accommodation of an emmetropic patient with a near point of 20 cm?
    1. 2 dioptres
    2. 5 dioptres
    3. 8 dioptres
    4. 10 dioptres
  6. What are the units of illuminance (luminous flux incident at a surface per unit area)?
    1. Candela
    2. Candela/m2
    3. Lux
    4. Watt/m2
  7. Regarding Jackson's cross cylinder:
    1. It is equivalent to two superimposed cylindrical lenses with axes at 45° to each other
    2. The cylinder in the cross cylinder has twice the power of the sphere
    3. The handle is traditionally in line with the axis of the negative cylinder
    4. When checking the power of the trial cylinder during refraction, the cross cylinder's handle is held in line with the axis of the trial cylinder
  8. Which of the following statements regarding the Prentice position is incorrect?
    1. Glass prisms are traditionally held in the Prentice position
    2. The angle of incidence equals the angle of emergence13
    3. The deviation of light is greater in the Prentice position than in the position of minimum deviation
    4. The incident ray is normal to one surface of the prism
  9. Regarding the reflection of light:
    1. A glass door appears transparent because no reflection of light occurs
    2. At a curved surface, the angle of reflection is equal to half the angle of incidence
    3. The incident ray, reflected ray and the reflecting surface are all in the same plane
    4. When a plane mirror is rotated by 90°, light reflected on the centre of rotation is deviated by 180°
  10. Which of the following best describes the image formed by reflection of an object at a plane mirror?
    1. Real, erect, laterally inverted
    2. Real, as far behind the surface as the object is in front of it, magnified
    3. Virtual, erect, diminished
    4. Virtual, erect, laterally inverted
  11. Regarding the refraction of light, which of the following statements is incorrect?
    1. The angle of refraction and the normal to the refracting surface both lie in the same plane
    2. The refractive index of the human crystalline lens is 1.386–1.406
    3. When entering an optically less dense medium, light is deviated away from the normal
    4. When light passes from air to Crown glass, the angles of incidence (i) and refraction (r) are related as follows:
  12. What is the spherical equivalent of −3.50/+1.50 axis 70?
    1. −2.00 DS
    2. −2.00/–1.50 axis 110
    3. −2.75 DS
    4. −5.00 DS/+1.50 DC axis 110
  13. During cataract surgery a complication occurs and the decision is made to place the intraocular lens (IOL) in the sulcus. A 23.5 dioptre IOL had been planned for placement in the capsular bag. Which of the following lens powers is most appropriate for placement in the sulcus?
    1. 21.5 dioptre
    2. 22.5 dioptre14
    3. 24.5 dioptre
    4. 25.5 dioptre
  14. Which of the following statements about the Maddox rod is false?
    1. A distant spot target is viewed with a Maddox rod over one eye only
    2. It is composed of parallel high-powered convex cylindrical lenses
    3. The glass of the Maddox rod is clear to eliminate chromatic aberration
    4. The line seen through the Maddox rod lies at 90° to the axis of the rod
  15. Which one of the following is not a test of stereoacuity?
    1. Cardiff cards
    2. Frisby
    3. Titmus
    4. Worth 4-dot
  16. Comparing direct and indirect ophthalmoscopy, which of the following statements is false?
    1. A patient's refractive error has a greater effect on indirect ophthalmoscopy
    2. Direct ophthalmoscopy gives greater magnification
    3. Indirect ophthalmoscopy gives an image that is inverted vertically and horizontally
    4. The field of view is greater with an indirect ophthalmoscope
  17. In simple hypermetropic astigmatism:
    1. Both focal lines are behind the retina
    2. Both focal lines are in front of the retina
    3. One focal line is on the retina whilst the other is behind it
    4. The focal lines are straddling the retina
  18. A patient's prescription is +2.00 DS/−1.00 DC axis 180. What is the toric transposition of this to the base curve −4 D?
    1. zoom view
    2. zoom view
    3. zoom view
    4. zoom view
  19. Which of the following indirect ophthalmoscopy lenses gives the widest field of view?
    1. 15 D
    2. 20 D15
    3. 28 D
    4. 30 D
  20. Which of the following best describes the image produced by a Galilean telescope?
    1. Virtual, erect, magnified, at infinity
    2. Virtual, erect, magnified, at the sum of the focal lengths of the objective and eyepiece lens
    3. Virtual, inverted, magnified, at infinity
    4. Virtual, inverted, magnified, at the sum of the focal lengths of the objective and eyepiece lens
  21. Regarding optical radiation, which of the following statements is false?
    1. Light emitted through fluorescence typically has a longer wavelength than the excitation light that triggers it
    2. Shorter wavelengths undergo increased scatter
    3. The energy of a photon is directly proportional to its wavelength
    4. The wavelength of light decreases as it enters a transparent material from a vacuum
  22. Which one of the following statements about interference is correct?
    1. Destructive interference is used in anti-reflection coatings
    2. Laser light generates less interference than conventional light sources
    3. The corneal stroma uses constructive interference in maintaining clarity
    4. Two waves of equal amplitude out of phase by one cycle will cancel each other out
  23. Regarding the prevalence of myopia (of at least −0.5 dioptres):
    1. It is high in white Europeans compared to other ethnicities
    2. It is lower in adults than in young children
    3. Myopia tends to decrease with nuclear sclerosis of the lens
    4. Time spent outdoors as a child is a protective factor against myopia
 
Pathology
  1. Which of the following tumours is not associated with neurofibromatosis?
    1. Acoustic neuroma
    2. Meningioma
    3. Phaeochromocytoma
    4. Renal cell carcinoma
  2. In which of the following conditions are Dalen–Fuchs nodules seen?
    1. Best disease16
    2. Coats disease
    3. Retinitis pigmentosa
    4. Sympathetic ophthalmia
  3. What percentage of uveal tract melanomas arise from the ciliary body?
    1. 12%
    2. 24%
    3. 36%
    4. 48%
  4. Which of the following statements about myasthenia gravis is false?
    1. An ice pack applied over the affected muscles for 5 minutes characteristically causes a temporary improvement in symptoms
    2. Autoantibodies are produced which target skeletal muscle acetylcholine receptors
    3. In Cogan's lid twitch, the upper lid overshoots when returning to the primary position from downgaze and there is a subsequent compensatory downward ‘twitch'
    4. There is characteristic muscle wasting, often involving facial and extraocular muscles
  5. Which of the following terms is used to refer to an increase in the number of cells in a tissue?
    1. Dysplasia
    2. Metaplasia
    3. Hyperplasia
    4. Hypertrophy
  6. Which of the following statements regarding amyloid is false?
    1. Amyloid appears as a homogenous blue material on haematoxylin and eosin (H&E) staining
    2. In lattice dystrophy, amyloid deposits occur in the corneal stroma
    3. Systemic amyloidosis is more common than localised amyloidosis
    4. When stained with Congo red, amyloid shows ‘apple’ green birefringence when exposed to polarised light
  7. Where in the retina are hard exudates found?
    1. Ganglion cell layer
    2. Nerve fibre layer
    3. Outer plexiform layer
    4. Between the photoreceptors and the RPE17
  8. Which of the following does not describe typical corneal arcus?
    1. Age-related
    2. Fatty degeneration
    3. Stromal lipid deposition
    4. Unilateral
  9. Regarding retinal detachment:
    1. Accumulation of subretinal fluid occurs in all types of retinal detachment
    2. Rhegmatogenous detachment is commonly inflammatory or vascular
    3. The retinal pigment epithelium is usually separated from the choriocapillaris
    4. The outer blood–retinal barrier is intact in serous retinal detachment
  10. Regarding proliferative vitreoretinopathy, which of the following is least correct?
    1. Contraction of fibrous membranes leads to retinal detachment
    2. Fibrosis and collagen production contribute to retinal membranes
    3. Raised intraocular pressure is common, due to inflammation
    4. RPE cell migration into the vitreous is characteristic
  11. A 30-year-old man sustains a full-thickness corneal laceration. Which of these is not part of the wound healing process?
    1. Fibrin plug formation and stromal oedema
    2. Regeneration of the epithelium and Bowman's layer
    3. Replacement of damaged stroma by scar tissue
    4. Transformation of aqueous-derived leukocytes into fibroblasts
 
Pharmacology
  1. Which of the following is not an effect of botulinum toxin A?
    1. Brief blockade of synaptic transmission
    2. Cleavage of SNARE proteins required for fusion of neurosecretory vesicles with the plasma membrane
    3. Inhibition of the exocytosis of acetylcholine at the neuromuscular junction
    4. Parasympathetic and motor nerve paralysis
  2. Regarding pharmacokinetics, which of the following statements is true?
    1. Eye drops absorbed in the systemic circulation via the nasopharyngeal mucosa undergo first-pass metabolism
    2. First-order kinetics are nonlinear
    3. Highly protein-bound drugs tend to distribute easily into tissues
    4. Lipid-soluble drugs can pass the blood–retinal barrier18
  3. Which of the following is not a side effect of ciclosporin?
    1. Bone marrow suppression
    2. Gingival hypertrophy
    3. Hirsutism
    4. Nephrotoxicity
  4. A patient with a known Pancoast tumour of the lung is referred to the eye clinic with anisocoria. This is more pronounced in darker conditions with the right pupil failing to dilate. Which of the following agents is likely to succeed in dilating both pupils?
    1. Cocaine 4%
    2. Hydroxyamphetamine 1%
    3. Pilocarpine 1%
    4. None of the above
  5. What is the mechanism of action of brimonidine?
    1. α2 agonist
    2. β antagonist
    3. Carbonic anhydrase inhibitor
    4. Prostaglandin analogue
  6. Which of the following is not a side effect of acetazolamide?
    1. Hyperkalaemia
    2. Metabolic acidosis
    3. Paraesthesia
    4. Renal stones
  7. Regarding eicosanoids, which of the following statements is true?
    1. Eicosanoids are derived from cyclooxygenase
    2. Nonsteroidal anti-inflammatory drugs (NSAIDs) inhibit the biological actions of eicosanoids on their target tissue
    3. NSAIDs reduce eicosanoid synthesis via inhibition of lipoxygenase
    4. Thromboxane A2 causes vasoconstriction
  8. Which of the following benzodiazepines has the shortest half-life?19
    1. Diazepam
    2. Lorazepam
    3. Midazolam
    4. Temazepam
 
Physiology
  1. Which of the following results from activation of histamine H1 receptors?
    1. Decreased intracellular calcium concentration
    2. Increased gastric acid production
    3. Increased vascular permeability
    4. Mast cell degranulation
  2. Which of the following statements regarding colour vision is true?
    1. Chromaticity depends on hue, saturation and luminance
    2. Green cones have maximum spectral sensitivity at 590 nm
    3. Hue is independent of luminance
    4. The term ‘colour’ refers to hue
  3. Which of the following statements about dark adaptation is true?
    1. Full dark adaptation takes approximately two hours
    2. In a dark adapted subject, a greater intensity of light is required to elicit a rod response than a cone response
    3. Increased cone sensitivity accounts for the early component of the dark adaptation curve
    4. Light adaptation is much slower than dark adaptation
  4. Which of the following statements regarding potassium is incorrect?
    1. Daily dietary potassium requirements are approximately 1 mmol/kg/day
    2. Insulin promotes the movement of potassium from the intracellular to the extracellular compartment
    3. Potassium is reabsorbed by the proximal convoluted tubule and secreted by the distal convoluted tubule of the nephron
    4. The concentration of potassium in intracellular fluid is 135–150 mmol/L
  5. Regarding the cardiac cycle, which of the following statements is false?
    1. Atrial depolarisation corresponds to the p-wave on an ECG
    2. Atrial contraction is responsible for 80% of ventricular filling
    3. Mitral valve closure precedes tricuspid valve closure
    4. Passive filling of the ventricles is rapid in early diastole
  6. Regarding the hypothalamic–pituitary axis, which of the following statements is incorrect?
    1. Gigantism is caused by over-secretion of growth hormone
    2. Somatostatin promotes the release of growth hormone from the anterior pituitary20
    3. The hypothalamic hormones are often referred to as ‘factors'
    4. The release of anterior pituitary hormones is pulsatile
  7. Which of the following processes is not thought to contribute to aqueous humour production?
    1. Active secretion
    2. Oncotic pressure
    3. Passive diffusion
    4. Ultrafiltration
  8. Neurones from the optic radiations terminate in which layer of the primary visual cortex?
    1. 1
    2. 2
    3. 3
    4. 4
  9. Which of the following statements about the organisation of the primary visual cortex is false?
    1. Cells from layer 6 provide ‘feed-forward’ innervation to the lateral geniculate nucleus
    2. Cells in layer 5 project principally to the secondary visual cortex
    3. Fibres representing the superior retina terminate in the superior lip of the calcarine sulcus
    4. Ocular dominance columns each respond to a single eye, alternating between columns
  10. Which of the following functions is thought to be directed by the suprachiasmatic nucleus?
    1. Auditory/visual integration
    2. Circadian rhythm
    3. Co-ordination of head and eye movements
    4. Pupillary light reflex
  11. Which glycosaminoglycan is thought to be most important in maintaining corneal clarity?
    1. Chondroitin sulphate
    2. Dermatan sulphate
    3. Heparan sulphate
    4. Keratan sulphate21
  12. Which of the following statements about action potentials in neurones is false?
    1. Action potentials are classically generated by a sudden influx of Na+ ions into the neurone
    2. Action potentials are triggered at the initial segment of the axon
    3. Between action potentials, neurones maintain a negatively charged resting membrane potential
    4. The magnitude of an action potential determines the quantity of neurotransmitter released
  13. Which of the following eye movements is disconjugate?
    1. Saccadic eye movements
    2. Smooth pursuit movements
    3. Vergence movements
    4. Vestibulo-ocular movements
  14. Which of the following is not a correct pair of yoked muscles?
    1. Left inferior oblique and right superior rectus
    2. Left medial rectus and right lateral rectus
    3. Right inferior oblique and left superior oblique
    4. Right superior oblique and left inferior rectus
  15. Which of the following is not a part of the accommodation response?
    1. Ciliary muscle relaxation
    2. Convergence
    3. Increase in lens curvature
    4. Miosis
  16. Regarding hearing, which of the following statements is incorrect?
    1. Depolarisation of the hair cell in the cochlea occurs due to the influx of sodium
    2. Higher frequencies are detected by hair cells near the base of the cochlea
    3. Sound is audible to humans between 20 and 20,000 Hz
    4. Sound is converted by the organ of Corti into action potentials in the auditory nerves
 
Statistics and evidence-based medicine
  1. A new test is being validated against the definitive gold standard test. 50 patients are tested positive by the gold standard test. Of these patients, 35 tested positive using the new test. A further five patients tested positive with the new test, but were found to be negative using the gold standard test. What is the sensitivity of the new test?22
    1. 50%
    2. 60%
    3. 70%
    4. 80%
  2. In a clinical study what is the correct definition of a type II error?
    1. Failing to reject the null hypothesis when it is false
    2. Failing to reject the null hypothesis when it is true
    3. Rejecting the null hypothesis when it is false
    4. Rejecting the null hypothesis when it is true23
 
Questions: CRQs
12 CRQs to be answered in 2 hours
 
Anatomy
  1. zoom view
    1. Identify the structures labelled A to G on the diagram showing a coronal section through the body of the sphenoid bone. (7 marks)
    2. Name two conditions affecting the venous system which typically affect this area of the body. (2 marks)
    3. Which cranial nerve is most commonly impaired in conditions affecting this area? (1 mark)24
 
Biochemistry and cell biology
  1. Below is a diagram of a sympathetic synapse at the dilator pupillae muscle.
    zoom view
    1. What are the microstructures labelled ‘A'? (1 mark)
    2. What is the principal neurotransmitter at this synaptic terminal? (1 mark)
    3. Which amino acid is this neurotransmitter synthesised from? (1 mark)
    4. Name two other neurotransmitters that are synthesised from this amino acid. (2 marks)
    5. What is the cellular process used to release neurotransmitters? (1 mark)
    6. What are the two types of receptor found at synapses, as defined by their mechanism of signal transduction, and which type is responsible for transmission at this synapse? (3 marks)
    7. What is the principal neurotransmitter at the synapse in the superior cervical ganglion? (1 mark)
 
Genetics
  1. A family comes to the ocular genetics clinic to discuss their condition. The proband is 32 years old. Ocular examination shows marked bilateral iris hypoplasia, nystagmus and reduced vision. She is registered partially sighted, works as a lawyer and is pregnant with her second child.
    The proband's father (aged 65) is affected and has bilateral absence of the iris, keratopathy, bilateral pseudophakia, nystagmus and is registered blind. Her mother (64) is unaffected. The proband has two unaffected siblings (male, 36; female, 29). Her youngest male sibling died in a car accident aged 23, but was also unaffected by the condition.25
    The proband has an affected daughter (14 months) with bilateral iris hypoplasia and nystagmus. The proband's husband has no eye conditions.
    1. Draw a pedigree (family tree) for this family. (5 marks)
    2. What is the likely mode of inheritance? (1 mark)
    3. What is the risk of her unborn child being affected? Please state whether the risk is affected by the sex of the child. (2 marks)
    4. State the most likely diagnosis. (1 mark)
    5. Which gene is most commonly responsible for this condition? (1 mark)
 
Investigations and imaging
  1. zoom view
    1. What is the name of this investigation? (1 mark)
    2. How is the image produced? (3 marks)
    3. Describe two limitations of this investigation. (2 marks)
    4. What is the principal abnormality in this scan? (1 marks)
    5. Give two possible diagnoses which could cause the above abnormality. (2 marks)
    6. What further retinal investigation could you order if the diagnosis was unclear? (1 mark)26
  2. The biometry below relates to one patient who is attending for consideration of cataract surgery (first eye).
    zoom view
    27
    zoom view
    1. Which of these two formulae is likely to be more accurate for the left eye, and which measurement allows you to determine this? (2 marks)
    2. Which two biometric measurements of the eye are used in both the SRK/T and Hoffer Q formulas? (2 marks)
    3. Which parameter is typically different when measured with contact ultrasound as opposed to immersion or optical biometry, and what causes the difference? (2 marks)
    4. What is the principal disadvantage of optical biometry? (1 mark)
    5. Where would you make your incision in the right eye if you wanted to reduce astigmatism? (1 mark)
    6. Refer to the SRK/T biometry. If the SN60WF IOL's A-constant is 119.0, and a further lens type has a predicted outcome of −0.09 for a 28.5 dioptre IOL in the right eye, what is the A-constant of this new IOL? (2 marks)28
  3. Below are images from two different investigations in the same eye of the same patient.
    zoom view
    1. What is the investigation labelled B above? (1 mark)
    2. In which part of the electromagnetic spectrum are the excitation source and fluorescence for investigation B? (2 marks)
    3. Give two reasons why the choroidal circulation cannot be well visualised with fluorescein angiography. (2 marks)
    4. In addition to providing better imaging of the choroidal circulation, give three further advantages of investigation B over fluorescein angiography. (3 marks)
    5. Why is the leakage of dye slower in investigation B than in fluorescein angiography? (1 mark)
    6. What is the mechanism of excretion of the dye used in investigation B? (1 mark)29
  4. Below are two axial CT images of the same patient from the same study.
    zoom view
    1. Describe how the images in a CT scan are obtained. (2 marks)
    2. What are the two principal abnormalities in image A? (2 marks)
    3. What further clinically significant abnormality can be seen in image B? (1 mark)
    4. What is the most likely diagnosis? (1 mark)
    5. What pupil abnormality would you look for and what is its significance? (2 marks)
    6. Name two possible intracranial complications of this condition. (2 marks)30
 
Microbiology
  1. A 36-year-old male recently returned from Bangladesh is seen in the eye casualty department with a corneal ulcer which developed following an abrasion. Microscopy of the scrape is shown below.
    zoom view
    1. What can be seen at A? (1 mark)
    2. What culture medium would be appropriate? (1 mark)
    3. What is the principal limitation of culture in this case? (1 mark)
    4. Name two specialist stains that could be used on this specimen in combination with a fluorescent microscope. (2 marks)
    5. Suggest two possible genera given this patient's history and corneal scrape. (2 marks)
    6. Name three risk factors for this form of keratitis. (3 marks)
 
Optics
  1. Right eye +1.50 DS/+1.50 DC axis 75° Left eye −1.50 DS/+0.75 DC axis 95°
    1. Express the above refractions in negative cylinder form. (2 marks)
    2. What is the spherical equivalent of the right eye? (1 mark)
    3. The patient is pseudophakic in the left eye but phakic in the right. She normally wears glasses. Name three options to address this lady's anisometropia. (3 marks)31
    4. If this patient develops increasing nucleosclerosis in her right eye, the anisometropia may decrease. Why is this? (1 mark)
    5. Describe two methods of reducing this lady's astigmatism during her right cataract surgery. (2 marks)
    6. When making a corneal incision, how will moving the incision site further from the limbus affect induced corneal astigmatism? (1 mark)
    1. Draw a diagram of Gullstrand's schematic eye, clearly labelling the first and second focal points; first and second principal points, and first and second nodal points. (6 marks)
    2. In which part of the eye is the nodal point located in the reduced schematic eye? (1 mark)
    3. Why does the cornea account for a greater proportion of the total refractive power of the eye than the lens, when the refractive index of the lens is greater than that of the cornea? (3 marks)
  2. Two patients attend the paediatric eye clinic. Orthoptic examination and cycloplegic refraction reveal the following:
    Patient 1, aged 6
    Visual acuity: 6/6 right eye, 6/7.5 left eye (Snellen acuity)
    Interpupillary distance: 55 mm
    Prism cover test (PCT): Near (33 cm or 3 D): +8 prism dioptres (∆) esophoria
    zoom view
    Cycloplegic refraction: +2.00 DS right eye, +2.50 DS left eye
    Patient 2, aged 4
    Visual acuity: 0.200 right eye, 0.150 left eye (logMAR acuity using Crowded Kay's pictures)
    PCT: esotropia (without glasses) measuring 30 ∆ for near and distance
    Accommodative convergence/accommodation (AC/A) ratio: 6
    Cycloplegic refraction: +5.00 DS in each eye
    1. What is meant by the term 'AC/A ratio'? (2 marks)
    2. What is the normal range for the AC/A ratio? (1 mark)
    3. Calculate the AC/A ratio for Patient 1, using the heterophoria method; show your working. (4 marks)
    4. Patient 2 is prescribed the full hypermetropic glasses correction. What effect will this have on the esotropia? Please quantify this and show your working. (3 marks)32
  3. zoom view
    1. What instrument is shown in part A? (1 mark)
    2. Please explain how prisms are used in taking a measurement. (2 marks)
    3. With the aid of a diagram, please explain the principle behind and forces involved in taking a measurement. (4 marks)
    4. In part B, which diagram shows that the force applied is too low? (1 mark)
    5. Please give two factors which may lead to an inaccurate measurement. (1 mark)
    6. How would interpretation of the test be affected in a patient with a corneal pachymetry reading of 620 μm? (1 mark)33
 
Answers: SBAs
 
Anatomy
  1. B Lacrimal nerve, frontal nerve, trochlear nerve
    The nerves passing outside the common tendinous ring are the lacrimal, frontal and trochlear nerves. The superior and inferior divisions of the oculomotor nerve, the nasociliary nerve and the abducens nerve pass within the common tendinous ring. There are several mnemonics based on the first letters of each nerve: LFT SNIA.
  2. D 11.7
    The diameter of the adult cornea is 11.7 mm horizontally and 10.6 mm vertically. At birth, the horizontal diameter is approximately 10 mm, with the adult size being reached by 2 years of age.
  3. B Sutures are mobile articulations
    The skull comprises separate bones which are joined by connective tissue known as sutural ligament. Sutures are immobile, rigid articulations. Diploë is the name of the spongy bone that separates tables of compact bone. The bones of the orbit include some bones of the cranium (frontal, sphenoid and ethmoid) and some of the facial bones (zygomatic, maxilla, palatine and lacrimal).
  4. B Middle meatus
    The paranasal sinuses all drain into the nose. Their opening sites into the nasal cavity are summarised in Table 1.1.
    Table 1.1   Opening sites of the paranasal sinuses
    Sinus
    Site of opening into nose
    Frontal
    Maxillary
    Ethmoidal (anterior and middle groups)
    Middle meatus
    Ethmoidal (posterior group)
    Superior meatus
    Sphenoidal
    Sphenoethmoidal recess or superior meatus
  5. C Sphenoid, ethmoid, lacrimal, maxilla
    The bones of the medial wall of the orbit are, from posterior to anterior: the body of the sphenoid, the orbital plate of the ethmoid, the lacrimal bone and the maxilla. These form the paper-thin separation between the orbit and the ethmoid sinuses. The lacrimal bone and maxilla form the lacrimal fossa, which houses the lacrimal sac.34
    Table 1.2 below summarises the bones of the four orbital walls. Note that the sphenoid forms part of all except the orbital floor. Whereas the roof and lateral wall are simpler with only two bony structures, some find the medial wall and floor easier to remember by their acronyms, SELM and ZyMP.
    Table 1.2   Bones of the four orbital walls
    Orbital wall
    Constituent bones
    Medial wall
    Sphenoid (body), ethmoid, lacrimal, maxilla
    Floor
    Zygomatic, maxilla, palatine
    Lateral wall
    Zygomatic, sphenoid (greater wing)
    Roof
    Frontal, sphenoid (lesser wing)
  6. B The Meibomian glands lie posterior to the grey line
    The Meibomian glands, also known as tarsal glands, are embedded in the tarsal plates. Their openings, 20–25 in each lid, are visible at the slit lamp posterior to both the lash line and the grey line. They are modified sebaceous glands which produce an oily substance that forms part of the precorneal tear film. This lipid layer reduces the rate of evaporation of tears. This explains why patients who are prone to blocked Meibomian glands, which is often called Meibomian gland dysfunction or posterior blepharitis, may experience dry eye symptoms. Anterior blepharitis involves crusting/debris and inflammation around the lash line, and the two often coexist.
    A chalazion is a lipogranulomatous inflammatory cyst resulting from a blocked Meibomian gland.
  7. A Parasympathetic fibres reach the lacrimal gland via the pterygopalatine ganglion and zygomaticotemporal nerve
    The lacrimal gland lies in the superolateral orbit and consists of an orbital and palpebral portion. It is important to read the question properly and avoid confusion with the lacrimal sac, which lies in the lacrimal fossa and is involved in drainage rather than production of tears.
    The innervation of the lacrimal gland is autonomic and sensory, as follows:
    • Parasympathetic supply, from the lacrimatory nucleus of the facial (seventh) nerve. These fibres travel via the nervus intermedius to the pterygopalatine ganglion, then via the maxillary nerve to its zygomatic and zygomaticotemporal branches, and finally via the lacrimal nerve to the gland. (Note that the maxillary and lacrimal nerves themselves are branches of the trigeminal nerve, but these parasympathetic fibres from the facial nerve nucleus are carried along with them).
    • Sympathetic supply, from the superior cervical ganglion. These fibres travel in the nerve plexus surrounding the internal carotid artery, then via the deep35 petrosal nerve to the maxillary nerve. From there, the course to the lacrimal gland is as for the parasympathetic fibres above.
    • Sensory supply, from the lacrimal nerve, which is a branch of the trigeminal nerve (V1).
  8. C The muscle fibres are more loosely packed, separated by connective tissue
    There are important differences between extraocular muscles and normal skeletal muscle found elsewhere in the body. The connective tissue of the extraocular muscles, including the epimysium (a sheath surrounding the whole muscle) is generally thin and delicate by comparison. The muscle fibres are more loosely packed, with the larger diameter (10–40 μm) fibres occupying the centre and the smaller fibres (5–15 μm) placed peripherally in the muscle. Extraocular muscle also has many specialised sensory and proprioceptive endings, including long muscle spindles almost 1 mm in length, which provide feedback about muscle activity to the central nervous system. Lastly, extraocular muscle is more vascular than normal skeletal muscle.
  9. A Meningeal branch of the facial nerve
    The facial nerve does not have a meningeal branch. The other three structures pass through the foramen spinosum, which connects the middle cranial fossa with the inferotemporal fossa.
  10. B Frontal nerve
    The frontal nerve is derived from the ophthalmic (V1) branch of the trigeminal nerve. There is a useful mnemonic for the branches of the facial nerve that has a number of variations. One publishable version is Two Zebras Bit My Coccyx:
    T – temporal nerve
    Z – zygomatic nerve
    B – buccal nerve
    M – mandibular nerve
    C – cervical nerve
  11. A Extradural haematoma
    The meningeal arteries, and in particular the middle meningeal artery and its dural branches, are vulnerable to injury from cranial trauma. This is particularly true in the temporal region, where the pterion is prone to fracture. As these vessels are extradural they typically result in an extradural haematoma (the pressure of arterial bleeding is required to create a plane between bone and dura).
  12. D Descemet's membrane
    Schwalbe's line marks the termination of Descemet's membrane. It is often visible clinically using gonioscopy and is visible as a smooth zone with scanning electron microscopy.36
  13. D Posterior pole
    The lens capsule is thinnest at the posterior pole (2–3 μm). The anterior pole is 9–14 μm and in the peri-zonular region the capsule is 17–28 μm thick. This is an important consideration during cataract surgery, particularly when approaching soft lens matter adherent to the posterior pole.
  14. C Peripheral retina and pars plana
    The vitreous base is a three-dimensional band that extends approximately 2 mm either side of the ora serrata and is usually attached to the peripheral retina and pars plana. It is firmly adherent, and clean dissection during vitrectomy surgery is very hard to achieve. It is extremely important that an adequate visualisation of the peripheral retina is achieved in cases of posterior vitreous detachment, as any resultant tears are likely to lie in this region.
  15. A Basement membrane of the choriocapillaris, outer collagenous layer, elastin layer, inner collagenous layer, basement membrane of the retinal pigment epithelium (RPE)
    Bruch's membrane is composed of 5 layers: the basement membrane of the choriocapillaris, an outer collagenous layer, an elastin layer, an inner collagenous layer and the basement membrane of the RPE.
  16. C Whitnall's tubercle
    The lateral canthal tendon inserts into Whitnall's tubercle, which is a small prominence of bone lying just inside the lateral orbital rim. Eisler's fat pad lies immediately anterior to the lateral canthal tendon, deep to the orbital septum, and is a useful surgical landmark for identifying Whitnall's tubercle.
  17. B The parasympathetic nerves to the iris synapse in the ciliary ganglion
    The parasympathetic nerves supplying the iris originate in the Edinger–Westphal nucleus and synapse in the ciliary ganglion. Their postganglionic fibres travel via the short ciliary nerves and supply the sphincter pupillae muscle. The dilator pupillae is predominantly innervated by unmyelinated sympathetic fibres originating in the superior cervical sympathetic ganglion. The sensory innervation of the iris is via both the long and short ciliary nerves, which derive from the nasociliary nerve (and ultimately V1).
  18. A Inferolateral
    From posterior to anterior the optic canal runs slightly inferiorly and around 36° laterally through the sphenoid. It is narrowest at the orbital opening and widens posteriorly. It connects the middle cranial fossa and the orbital apex.37
  19. B The ciliary body has a greater nasal anteroposterior length than temporal anteroposterior length
    The ciliary body has a greater anteroposterior length temporally than nasally, and is generally at its longest inferotemporally. It varies with the size of the globe, measuring 5.6–6.3 mm temporally and 4.6–5.2 mm nasally. The other statements are true. The third (innermost) muscle layer is composed of circular fibres. The ciliary body stroma lies between the ciliary muscle and the ciliary epithelium.
  20. B Most of the anastomoses between the major and minor arterial circles run through the anterior border layer
    The anterior border layer of the iris normally contains very few blood vessels, and significant vascularisation suggests ischaemic insult. There are extensive anastamoses between the major and minor arterial circles which run predominantly through the iris stroma in a radial manner. Iris capillaries are non-fenestrated with tight junctions and pericytes, contributing to the blood–ocular barrier. The major arterial circle (which is often incomplete) derives its blood supply from the anterior ciliary arteries (which travel anteriorly with the extraocular muscles before piercing the sclera) and the long posterior ciliary arteries (which pierce the sclera near the optic nerve and run anteriorly between the sclera and choroid to the ciliary body). The minor arterial circle of the iris is at the level of the collarette, and is also often incomplete. From here vessels continue inwards to the pupillary border.
  21. D Right subclavian vein
    The lymphatic ducts from the right side of the head and neck, and the right arm, drain into the right subclavian vein. The final common lymphatic duct before entering the venous system is the right lymphatic duct.
    The thoracic duct collects lymph from the rest of the body and drains into the left subclavian vein.
    The blood from the subclavian veins returns to the heart via the brachiocephalic veins and superior vena cava.
  22. B The internal elastic lamina is in the intima
    The three principal layers (tunica) of blood vessels, from the lumen outwards, are as follows:
    1. Intima
    2. Media
    3. Adventitia
    The intima contains the endothelium supported by the internal elastic lamina. The latter is typically fragmented in giant cell arteritis.
    The media is the main muscular layer of the vessel, containing smooth muscle cells, and varying amounts of elastic fibres (principally in medium to large arteries).38
    The adventitia (sometimes called tunica externa) is the main connective tissue layer, and in arteries this contains the external elastic lamina.
    Veins have a thinner media, larger lumen and contain proportionally less muscle and less elastic tissue than arteries.
 
Biochemistry
  1. B Glucose
    Compared to plasma, aqueous contains:
    • Higher levels of lactate and ascorbate;
    • Similar concentrations of sodium, potassium and magnesium;
    • Slightly lower levels of bicarbonate;
    • Lower levels of glucose and calcium (about half that of plasma), and much lower levels of albumin.
  2. B Glycosylation of collagen is vitamin C-dependent
    The steps of collagen synthesis are summarised in Table 1.3. The end result is an insoluble, extracellular glycoprotein.
    Table 1.3   Steps of collagen synthesis
    Stage of collagen synthesis
    Further details
    Transcription
    DNA → RNA
    Translation
    RNA → peptide formation
    The basic amino acid sequence is repeating triplets (Glycine-X-Y)n
    where X is often proline, Y is often hydroxyproline
    Polypeptides (primary structure) twist into a left-handed helix (secondary structure; ≠ α-helix)
    Post-translational modification (endoplasmic reticulum)
    Vitamin C-dependent hydroxylation of the Y-position amino acid Glycosylation
    Triple helix formation (tertiary structure) linked by covalent S-S bonds
    Excretion from the cell
    Cleavage of terminal peptide chains, making it insoluble
    Extracellular modification
    Lysine oxidation
    Formation of crosslinks
  3. A Endogenous inhibitors called TIMPs are present in normal tissues
    Matrix metalloproteinases (MMPs) are zinc and calcium-dependent endopeptidases which act as an enzyme cascade to degrade extracellular matrix. They can potentially degrade all types of extracellular matrix, and include collagenases and gelatinases. MMPs are released by neutrophils during the acute inflammatory response (they mediate cell and tissue damage, along with free radicals from the respiratory burst), and also by cells in normal tissues for growth, maintenance, repair and remodelling. For example, when stimulated by growth factors, one of39 the responses of a cell is to release MMPs to degrade the surrounding extracellular matrix to create space for the increase in cell numbers. They are kept in check by endogenous tissue inhibitors of MMPs (TIMPs).
  4. B Epithelial growth factor stimulates the differentiation of lens epithelial cells into lens fibres
    One third of the weight of the lens is protein, and 90% of the lens protein is crystallins. Secondary lens fibres are formed from the anterior lens epithelium. Fibroblast growth factor (FGF) is one of the principal factors that stimulates the differentiation of anterior lens epithelial cells into lens fibres. This change is characterised by elongation, withdrawal from the cell cycle, expression of large amounts of crystallins and eventually the loss of organelles and the nucleus.
    Lens fibres are densely packed with cytoplasmic interdigitations, and intercellular, gap junction-like channels formed by MIP26 (main intrinsic polypeptide 26), which was subsequently recognised as an aquaporin and named aquaporin-0 (AqP0).
    The genes encoding the two types of α-crystallin (known as αA and αB) are found on chromosomes 21 and 11, respectively.
  5. D Glycosaminoglycans have a high positive charge
    Glycosaminoglycans have a high negative charge. They are polysaccharides composed of long chains of repeating disaccharides. Unlike proteins, they are inflexible. As highly charged molecules they are strongly hydrophilic, and form a gel that is resistant to compression. With the exception of hyaluronan they may be found as sulphated molecules, and may be attached to a protein, forming a proteoglycan. The functions of these proteoglycans are extremely diverse.
  6. B There is active transport of sodium into the aqueous
    The corneal endothelial cell pump moves sodium and bicarbonate from the stroma to the aqueous by active transport. This is mediated by a sodium/potassium-dependent ATPase and a bicarbonate-dependent ATPase, and is facilitated by co-transporters. Potassium and chloride move to the aqueous predominantly by passive diffusion.
  7. B Vitamin C
    Vitamins A, D, E and K are lipid soluble, whereas vitamins B and C are water soluble. Lipid soluble vitamins can be stored in the liver and adipose tissue, whereas water soluble vitamins must be consumed more regularly. Patients with either reduced lipid intake or reduced lipid absorption may develop deficiencies of lipid soluble vitamins, as they are normally absorbed in chylomicrons.
  8. C Pericyte coverage is highest in the choroid
    Pericyte coverage is highest in the retinal vessels, with a relative frequency of 1:1 pericytes to endothelial cells. This may be because of the meticulous metabolic demands of the retina and the requirement to maintain the blood–retinal barrier.40
    Conversely the choroid vessels have a much less dense coverage of pericytes, facilitating exchange of metabolites. Pericytes are contractile cells embedded in the basement membrane of vessels, to which they contribute. They are involved in a number of regulatory functions, communicating with the endothelial cells by paracrine signalling and direct contact. They play a key role in maintaining the blood–brain barrier, where they regulate permeability. They may also have a role in controlling cerebral blood flow.
  9. A Aflibercept is an anti-VEGF monoclonal antibody
    Aflibercept is a fusion protein acting as a decoy receptor molecule for VEGF (it contains binding domains from type 1 and type 2 VEGF receptors). Bevacizumab is a monoclonal antibody to VEGF, and ranubizumab is a monoclonal antibody fragment from the same mouse antibody. VEGF synthesis is upregulated by a large number of metabolic and growth factors including TNFα, TNF-β, IL-1α, IL-6, prostaglandin E2, epidermal growth factor, fibroblast growth factor, insulin-like growth factor-1, keratinocyte growth factor and platelet-derived growth factor. All three identified VEGF receptors act via intracellular tyrosine kinase activity. In addition to promoting the division and survival of vascular endothelial cells, VEGF promotes vascular dilatation and permeability to macromolecules. It also has functions in regulating gene expression, cell migration and nitric oxide production.
  10. C Rod outer segments complete their renewal cycle every 21 days
    About 10% of the rod outer segment is phagocytosed daily, and it takes 9–10 days for the renewal cycle to be complete. Melatonin is synthesized by photoreceptors in conditions of darkness and suppresses dopamine production. Conversely, local dopamine production increases during conditions of light and suppresses melatonin production. These mechanisms are involved in modulating the circadian rhythm of photoreceptor renewal, with rod disc shedding and phagocytosis occurring at first light. Cone outer segments are predominantly shed at the onset of darkness. Rod discs are phagocytosed in groups of approximately 200. It is hard to measure cone segment phagocytosis experimentally.
 
Embryology, growth and development
  1. D Surface ectoderm
    The lens originates from a thickening of surface ectoderm called the lens placode, which invaginates to form the lens vesicle. By day 36 this separates from the remaining surface ectoderm. The surrounding basal lamina will later form the lens capsule.
  2. A Inferonasal
    Colobomas are a result of failed or incomplete closure of the embryonic optic fissure, and are thus found inferonasally. The fissure is important for allowing the hyaloid artery to be incorporated into the developing globe.41
  3. B Second pharyngeal arch
    The orbicularis oculi muscle, like all muscles of facial expression, is derived from the second pharyngeal arch. The muscular contributions of the pharyngeal arches are summarised in Table 1.4.
    Table 1.4   Muscular contributions of the pharyngeal arches
    Pharyngeal arch
    Muscular contributions
    1
    Muscles of mastication (masseter, pterygoids, temporalis); mylohyoid; tensor tympani; tensor veli palatini; anterior belly of digastric
    2
    Muscles of facial expression (surrounding the eye these include orbicularis oculi; corrugator supercilii; depressor supercilii; procerus and occipitofrontalis); stylohyoid; stapedius; posterior belly of digastric
    3
    Stylopharyngeus
    4
    Pharyngeal constrictors, levator veli palatini, cricothyroid
    6
    Intrinsic muscles of the larynx except cricothyroid
  4. D Month 7
    Axons grow from developing retinal ganglion cells, changing course at the optic disc and growing towards the optic stalk and brain.
    Optic nerve fibres are present from week 6 to 7. Myelination starts from the optic chiasm in month 7 of gestational development, but continues after birth. It is complete by 1 to 3 months of age in a full term baby.
  5. D The vitreous liquefies first in the periphery causing the posterior vitreous cortex to detach
    In childhood, type II collagen and hyaluronate gradually increase in concentration in the vitreous, which is homogeneous without visible fibrils. In adulthood, fibrils become visible due to the dissociation of hyaluronate and collagen, and aggregation of the latter.
    The central vitreous is the first to liquefy. The vitreous reduces in size, and liquefied vitreous (from pooling of hyaluronate) escapes into the subhyaloid space, leading to posterior vitreous detachment.
 
Genetics
  1. C Klinefelter syndrome
    Klinefelter syndrome is caused by an additional X chromosome, rather than an additional autosomal chromosome (trisomy 21 for Down syndrome; trisomy 18 for Edwards syndrome; trisomy 13 for Patau syndrome).42
  2. A Introns are removed from the RNA strand before it leaves the nucleus
    Introns are removed from the mRNA strand and the remaining exons are spliced together before the molecule leaves the nucleus. The enzyme primarily responsible for creating the resultant RNA molecule is RNA polymerase. This binds to the template strand of DNA which is copied to make a mirror RNA strand. The coding strand of DNA is the opposite strand of DNA which therefore corresponds directly to the resultant RNA sequence. RNA polymerase reads the DNA strand in a 3’ to 5’ direction, and therefore extends the resultant RNA strand in a 5’ to 3’ direction.
  3. C Mitochondrial
    As only the mother's mitochondria are passed on, the offspring of an affected male will not inherit the condition. Another mitochondrial-inherited condition with ophthalmological manifestations is Kearns–Sayre syndrome.
  4. B Chimeras have genetically distinct cell populations arising from one zygote
    The terms mosaic and chimera apply to an organism composed of cells of two or more genotypes. In mosaicism, the cells all arise from the same zygote. In chimerism, the cells arise from two or more zygotes.
    Somatic mosaicism occurs naturally in all women due to the process of X-inactivation (lyonisation), where one X-chromosome of their somatic cells is randomly inactivated. It occurs by day 12 of embryonic development, with all descendants of that cell having the same functional X-chromosome (either maternal or paternal). This can give rise to patchy expression of mutant X-linked genes. Somatic mosaicism also occurs in cancer cells, with mutations in the proliferating tumour cell line that are not found in normal cells.
    The term chimera can be applied more broadly in science, for example to proteins (Chimera is the name of a mythical ancient Greek creature that was part lion, part goat and part serpent). Some monoclonal antibodies are chimeric fusion proteins. For example, murine B cells producing antibodies against a desired target can be genetically engineered to introduce human constant domains, reducing the chance of immune rejection. Monoclonal antibodies whose name ends –ximab fall into this category (e.g. infliximab), whereas ‘fully humanised’ monoclonal antibodies end in –zumab.
  5. D The probability of crossing over between two loci on the same chromosome is proportional to the distance between them
    If two genetic loci are on different chromosomes, they will segregate independently when gametes are formed by meiosis. The converse is not true: if they are on the same chromosome, they do not always segregate together due to the fact that crossing over (recombination) occurs between non-sister chromatids during prophase I of meiosis.43
    The technical aim of genetic linkage analysis is to discover how often two loci are separated by meiotic recombination. The probability of crossing over between two genetic loci on the same chromosome is proportional to the distance between them.
    Families undergoing linkage analysis need to express a clear phenotype. By establishing genetic markers of known loci with the affected and nonaffected phenotypes, linkage analysis can provide information about the genetic status of family members at risk of the condition. Highly polymorphic genetic loci are the most informative for linkage analysis, as they are heterozygous in a large proportion of the population.
    Linkage is particularly useful in cases where the exact mutation cannot be identified in a family, so direct DNA analysis is not possible.
  6. D Histones have a high negative charge
    Histones have a high positive charge, allowing them to associate with DNA which has a negative charge. Lysine and arginine are positively charged (basic) amino acids, and are found in large numbers in histones. Histones form octamers which form the core of each nucleosome. DNA is wound round the histone core. Histones can be dynamically modified by a number of post-translational modifications, occurring predominantly on the accessible histone ‘tails'. These modifications alter histone–DNA binding and influence gene expression.
  7. B DNA polymerase β is the principal enzyme in mitochondrial DNA replication
    DNA polymerase γ is the principal enzyme in mitochondrial DNA replication. The alpha subunit of this enzyme is encoded by the POLG gene on chromosome 15. Mutations in POLG can lead to a number of mitochondrial diseases including progressive external ophthalmoplegia (both autosomal dominant and autosomal recessive forms) and Alpers’ syndrome. The circular double-stranded DNA of mitochondria encodes 22 tRNAs, 2 rRNAs and a number of polypeptides required for oxidative phosphorylation.
  8. D 50%
    Reis–Buckler corneal dystrophy is autosomal dominantly inherited and so the risk to offspring is 50%. Apart from macular corneal dystrophy, which is autosomal recessive, it is safe to assume that a corneal dystrophy is autosomal dominant.
    Reis–Buckler is characterised by granular deposits in the epithelial basement membrane. These stain with Masson's trichrome. It manifests clinically as recurrent corneal erosion and corneal opacification which may require corneal grafting.44
  9. D Ribozymes
    Gene therapy aims to treat a genetic condition by replacing the abnormal gene with a healthy one or by manipulating its expression.
    Vectors are used to insert the DNA into the chromosome of the cell, and these can be divided into viral and non-viral vectors. Viral vectors exploit the normal viral transfection/transduction abilities to deliver the desired gene, and include adenoviruses, retroviruses, adeno-associated viruses and herpes simplex viruses. Non-viral vectors include liposomes and naked DNA in the form of plasmids.
    Ribozymes are also a tool in gene therapy but they are not vectors: they are used to cut mRNA of the target gene to block transcription.
 
Immunology
  1. B Mucosa-associated lymphoid tissue (MALT)
    The main components of the innate immune system are as follows:
    1. Physical: physicochemical barriers, e.g. eyelids, tears, skin
    2. Chemical:
      1. molecules present in body fluids, e.g. complement, antiproteases, C-reactive protein, lysozyme;
      2. molecules released by cells when needed, such as cytokines (e.g. interferon and TNFα), unless they are released by antigen-specific cells in which case they would fall under the acquired immune system
    3. Cells: phagocytic and cytotoxic cells, e.g. macrophages, neutrophils and NK cells
    MALT, which contains lymphocytes, is a component of the acquired immune system. The principal characteristics of the innate, compared to the acquired immune system, are that it is more rapid and less specific. Whilst the acquired immune system generates receptors specific to a particular epitope, the innate immune system is either non-specific in its response, or broadly specific. An example of the latter is the receptors on cells such as phagocytes which recognise pathogen-associated molecular patterns (PAMPs). PAMPs are well-conserved molecular structures that are often shared by related pathogens, such as the flagellin of bacterial flagella.
    There is a lot of interaction between the innate and acquired immune systems; for example IgM antibodies are potent activators of the complement system.
  2. D T cells
    Antigen presenting cells (APCs) include: monocytes, macrophages, dendritic cells, some B cells, and activated endothelial cells. They constitutively express MHC class II molecules.45
    APCs do not include T cells; the main function of APCs is to allow T cells to recognise antigens.
  3. D Viral proteins presented with MHC class I molecules are recognised by CD4+ T cells
    T cells do not respond to peptide antigen that is free in solution. For a T cell to recognise an antigen, the antigen must be presented on the cell surface in association with MHC molecules.
    MHC class I molecules are present on every nucleated cell, whereas MHC class II molecules are only expressed by antigen presenting cells (APC). HLA-B27 is an example of an MHC class I molecule.
    Intracellular antigens such as viral proteins are complexed with MHC class I molecules and presented to CD8+ T cells. By contrast, extracellular antigen (which has been digested by the APC into short peptides) is presented with MHC class II molecules for recognition by CD4+ cells.
 
Investigations and imaging
  1. C Optic neuritis
    From an electrical point of view, the eye functions as a dipole. The electro-oculogram (EOG) measures the resting electrical potential between the retinal pigment epithelium (electrically negative) and cornea (positive). It is tested under both dark- and light-adapted conditions and requires a cooperative patient who can make horizontal eye movements.
    It is essentially a test of the function of the outer retina and retinal pigment epithelium (RPE), so is abnormal in conditions with a generalised disruption of the RPE-photoreceptor interface or in conditions with generalised RPE disease.
    The EOG is normal in optic nerve disease. Two classic examples of diseases in which the EOG is abnormal are Best's vitelliform macular dystrophy and acute zonal occult outer retinopathy (AZOOR).
  2. D Fat
    MRI scanning has a number of different imaging sequences, and in particular it is important to know the distinction between T1-weighted and T2-weighted images, the two main sequences. Key differences in tissue appearance are summarised in Table 1.546.
    Table 1.5   Tissue appearances on MRI: T1 versus T2
    Tissue
    T1-weighted MRI
    T2-weighted MRI
    Air
    Black
    Black
    Dense bone
    Black
    Black
    Calcium
    Black
    Black
    Low protein fluid (e.g. cerebrospinal fluid)
    Dark
    Bright
    White matter
    Grey
    Dark
    Grey matter
    Grey
    Grey
    Acute blood
    Grey
    Dark
    Subacute blood
    Bright
    Bright
    Fat
    Bright
    Moderately bright
    High-protein fluid (e.g. highly concentrated mucus)
    Bright
    Dark
 
Microbiology
  1. A Bacillus cereus is frequently implicated in chronic low-grade endophthalmitis
    Bacillus cereus causes a rapidly progressing endophthalmitis which is devastating to the ocular tissues, causing advanced tissue damage within 12–24 hours.
    Coagulase-negative staphylococci have been isolated in nearly 70% of cases of endophthalmitis post-cataract surgery in several studies.
    Endogenous endophthalmitis is rare, accounting for fewer than 10% of all cases of endophthalmitis.
    Pseudomonas species produce a number of enzymes which promote tissue destruction and limit the efficacy of some antimicrobials.
  2. A Löwenstein–Jensen medium
    Löwenstein–Jensen medium is optimised to culture Mycobacterium tuberculosis, while inhibiting the growth of other bacteria. This is necessary due to the slow doubling time of M. tuberculosis (15–20 hours) and resultant long incubation. Sabouraud's agar is used to culture fungi and filamentous bacteria. Theyer–Martin medium is useful for isolating Neisseria species. MacConkey agar is used to grow cultures of Gram-negative bacteria, and to determine whether they are lactose fermenting.
  3. C Rifampicin
    Rifampicin inhibits bacterial DNA-dependent RNA polymerase, blocking the synthesis of bacterial RNA. Chloramphenicol binds to the 50s subunit of the bacterial ribosome, inhibiting bacterial protein synthesis. Vancomycin and cefuroxime inhibit bacterial cell wall synthesis.47
  4. D 180 minutes
    Dry heat sterilisation takes considerably longer than moist heat sterilisation. Exposure time is inversely related to the temperature of exposure. Minimum sterilisation times for dry and moist heat are shown in Table 1.6 (as per World Health Organization guidelines).
    Table 1.6   Minimum sterilisation times for dry and moist heat
    Moist Heat
    Temperature (°C)
    Minimum Sterilisation Time (min)
    121-124
    15
    126-129
    10
    134-138
    5
    Dry Heat
    Temperature (°C)
    Minimum Sterilisation Time (min)
    160
    180
    170
    60
    180
    30
  5. C It can be grown on Escherichia coli nutrient-deficient agar
    Acanthamoeba may be grown on E. coli nutrient-deficient agar from samples including corneal scrapes and contact lens cases. Acanthamoeba keratitis is characterised by pain in excess of what might be expected from clinical findings. While Acanthamoeba keratitis is more common in contact lens wearers than the general population it is still far less common than bacterial keratitis. Acanthamoeba exists both as trophozoites and as cysts. Trophozoites are relatively sensitive to treatment, but cysts can take many weeks to eradicate.
  6. C Inhibition of acetylcholine exocytosis
    Botulinum toxin A is a protein produced by Clostridium botulinum, a Gram-positive bacteria. It is internalised by cholinergic nerve terminals and inhibits the exocytosis of acetylcholine.
  7. B Cytomegalovirus
    Cytomegalovirus (CMV) typically causes uveitis (anterior and/or posterior). CMV retinitis is a common cause of visual loss in immunocompromised individuals. Whilst CMV is a recognised cause of keratitis, in the form of corneal endotheliitis, keratitis is a much more common presentation of the other three viruses listed.
    Viral keratitis is often associated with inflammation in other ocular structures. For example adenovirus causes keratoconjunctivitis, and herpes zoster (varicella zoster virus) causes keratouveitis. Herpes simplex is a common cause of recurrent keratitis,48 which may affect the corneal epithelium (e.g. dendritic ulceration), stroma or endothelium, giving rise to different clinical pictures.
  8. B Leukocoria
    Toxocariasis is a nematodal infection that typically affects children. It causes posterior uveitis and is a part of the differential diagnosis of leukocoria. Both toxocara and toxoplasma can be transmitted to humans via ingestion of animal faeces. Toxocara canis ova are shed in dog faeces; Toxocara cati ova and Toxoplasma gondii cysts are shed in cat faeces. Both toxocara and toxoplasma cause white chorioretinal lesions and, in some cases, vitritis. Infection may be asymptomatic.
  9. A It is a cause of orbital cellulitis
    Haemophilus influenzae is an aerobic, Gram-negative bacillus. The descriptor coccobacillus is often applied, as the rods are very small and can give the appearance of cocci. Haemophilus influenzae is best cultured on chocolate agar with added X and V factors. The organism was believed to be responsible for influenza, before this was established to be a viral infection.
    It is a common cause of respiratory tract infections and sinusitis, the latter becoming orbital cellulitis if spread through the orbital wall occurs.
    The virulence of the serotype is related to the capsule, with encapsulated serotypes being more virulent and invasive. Encapsulated strains are classified on the basis of capsular antigens into serotypes a–f. Vaccination is only routinely available against Haemophilus influenzae type b, a serotype associated with meningitis particularly in young children. The Hib vaccine, as it is known, is part of the routine vaccination schedule in the UK.
 
Optics
  1. A × 4
    Magnification power of a loupe is given by the equation:
    zoom view
    where M is the magnification and F is the power of the lens in dioptres. This is derived using the standard near point of an adult eye, accepted as 25 cm (1/4 m).
  2. A Real, inverted, diminished
    Objects outside the centre of curvature of a concave mirror will produce an image that is real, inverted and diminished, lying between the centre of curvature and the principal focus. It is always worth drawing the ray diagram for these questions. One ray runs from the top of the object parallel to the principal axis, and is reflected through the principal focus. The other runs from the top of the object through49 the centre of curvature and will therefore hit the mirror at 90° to the surface and be reflected along the same path. The image is formed where these two rays intersect. This is demonstrated in Figure 1.1, where O represents the object position, I represents the image position, C represents the centre of curvature, and F represents the principal focus.
    zoom view
    Figure 1.1:
  3. A 2°
    The angle of deviation in a prism in the position of minimum deviation is defined by the formula:
    zoom view
    where D is the angle of deviation, n is the refractive index and α is the apical angle.
    Crown glass has a refractive index of 1.52, so for any crown glass prism in air:
    zoom view
    50
  4. B Erect, virtual, further from lens than object, magnified
    An object located within the first focal point of a convex lens will produce an image that is erect, virtual, further from the lens than the object and magnified. It is always worth drawing the ray diagram for these questions. One ray runs from the top of the object parallel to the principal axis, and is refracted through the second principal focus (note that in the case of concave lenses it is a virtual ray that passes through the second principal focus). The other runs from the top of the object through the principal point at the centre of the lens, and continues undeviated. The image is formed where these two rays intersect. This is demonstrated in Figure 1.2, where O represents the object position, I represents the image position, F1 represents the first principal focus and F2 represents the second principal focus.
    zoom view
    Figure 1.2:
    51
    zoom view
    Figure 1.2:
  5. B 5 dioptres
    Amplitude of accommodation is given by the formula:
    zoom view
    where A is the amplitude of accommodation, P is the near point in dioptres and R is the far point in dioptres.
    The dioptric power can be calculated as the reciprocal of the distance in metres. In this case:
    zoom view
    The far point in emmetropia is infinity, and for these purposes the reciprocal of infinity is taken as 0. Therefore, 5 − 0 = 5 dioptres of accommodation.
  6. C Lux
    Lux are photometric units of illuminance, measured as lumen/m2. Watt/m2 are the equivalent radiometric units of irradiance.
  7. B The cylinder in the cross cylinder has twice the power of the sphere
    Jackson's cross cylinders are made up of a cylinder that is twice the power of the sphere, and of the opposite sign. Another way of describing it is a pair of cylindrical lenses with axes at 90° to each other. The handle is positioned between the two axes, so lies at 45° to each axis.
    The cross cylinder allows testing of the axis and power of the cylindrical element during subjective refraction. Whilst checking the axis, the handle of the cross cylinder is held in line with the axis of the trial cylinder. Whilst checking the power, the positive or negative cylinder marking on the cross cylinder is held in line with the axis of trial cylinder. When no cylindrical element has been found on retinoscopy, the cross cylinder can also be used to confirm this.52
  8. B The angle of incidence equals the angle of emergence
    In the Prentice position, a prism is placed such that the light ray is normal to one surface of the prism. All the deviation occurs on the far surface (Figure 1.3). Unlike in the position of minimum deviation, the angle of incidence does not equal the angle of emergence.
    The Prentice position power is typically specified for glass prisms, whereas plastic ones are usually intended to be used in the position of minimum deviation. If a prism is held in the wrong position, its effective power changes.
    zoom view
    Figure 1.3:
  9. D When a plane mirror is rotated by 90°, light reflected on the centre of rotation is deviated by 180°
    The laws of reflection are as follows:
    • The angle of incidence equals the angle of reflection. Both angles are usually measured with respect to the normal to the reflecting surface.
    • The incident and reflected rays, as well as the normal to the reflecting surface, are all in the same plane.
    These are true regardless of the nature of the surface.
    Some reflection of light occurs at all interfaces, even if most of the light is transmitted, such as with a glass door.
    If light is reflected by a plane mirror that is rotated, the reflected ray is deviated by an angle twice as large as the angle of rotation. This is because both the angles of incidence and reflection are increased by the angle of rotation.
  10. D Virtual, erect, laterally inverted
    Reflection of an object at a plane (flat) mirror produces an image that is virtual, erect and laterally inverted (Figure 1.4). The object is in front of the mirror, and the image lies the same distance behind it, on a line perpendicular to the mirror. The image is neither enlarged nor diminished.53
    zoom view
    Figure 1.4:
  11. D When light passes from air to Crown glass, the angles of incidence (i) and refraction (r) are related as follows:
    zoom view
    The correct relationship is:
    zoom view
    When passing into an optically denser medium, light is deviated towards the normal (the normal to the surface at the point of incidence). Conversely, when light enters an optically less dense medium, it is deviated away from the normal.
    Snell's law describes the relationship between the optical densities of the two media and the angles of incidence (i) and refraction (r):
    zoom view
    where n1 is the refractive index of the first medium, and n2 the second medium; and where the incident ray, refracted ray and the normal are all in the same plane.
    When light passes from air (refractive index of 1) into another medium, the equation becomes:
    zoom view
    The example in option (d) in the question, air to Crown glass (refractive index of 1.52), is therefore false because it should be as follows:
    zoom view
    54
  12. C -2.75 DS
    The spherical equivalent is the closest spherical lens power to a given toric lens. It is calculated by adding half the cylindrical power to that of the sphere.
    In the example given, the spherical lens power is −3.50 with a +1.50 cylinder at axis 70. The spherical equivalent of this is therefore:
    zoom view
    Option B is the correct transposition of this lens from positive to negative cylinder format, rather than the spherical equivalent.
  13. B 22.5 dioptre
    As a lens placed in the sulcus has effectively been moved further from the retina its effective power will increase, and so the power of the intraocular lens (IOL) must be reduced to compensate. Studies have shown that an adjustment of −1 dioptres is appropriate in eyes with a normal axial length. If the IOL power predicted for the capsular bag is greater than 28 dioptres then an adjustment of −1.5 would be appropriate. Similarly, if the IOL power predicted for the capsular bag is less than 17.5 dioptres then an adjustment of −0.5 would be appropriate. For very low IOL powers (below 9.5 dioptres) the IOL power should not be changed.
  14. C The glass of the Maddox rod is clear to eliminate chromatic aberration
    The glass of the Maddox rod is tinted (usually red) in order to give a coloured line to compare to the white dot seen with the other eye.
    A white spot target is viewed with a Maddox rod over one eye. This should be at least 6 metres distant.
    A line is seen through the Maddox rod at 90° to the axis of the rod. This is because the light parallel to the axis of the Maddox rod is unaffected by it, and therefore is focused on the retina normally forming a row of foci at 90° to the axis (the row of foci formed are perceived as a line). Conversely the light at 90° to the axis is focused by the powerful convex cylindrical lenses of the rod and form a series of lines immediately in front of the eye that run parallel to the axis of the Maddox rod. The light from these lines is dispersing at a sharp angle when it reaches the cornea and it is thus out of focus at the retina.
  15. A Cardiff cards
    The Cardiff cards test is a preferential looking test of visual acuity aimed at toddlers aged 1–3 years, although it can also be used in older children or adults with intellectual impairment.
    Frisby testing involves identifying the ‘different’ picture from a group of patterns on varying thicknesses of plate (one will have a hidden circle printed on the back surface only visible with stereopsis). It does not require special glasses, and can be used in young children.55
    The Titmus test (sometimes called the Wirt fly test as one of the key images is a housefly) involves looking at vectographs through polarised glasses. The composite image is seen if sufficient stereoacuity is present.
    The Worth 4-dot test involves viewing four dots arranged in a diamond: two green, one red, and one white. Red/green goggles are worn. The number and position of the lights seen allow assessment of stereopsis and binocular function.
  16. A A patient's refractive error has a greater effect on indirect ophthalmoscopy
    A patient's refractive error has a greater effect on direct ophthalmoscopy. Direct ophthalmoscopy gives greater magnification (x15 in the emmetropic eye) than indirect ophthalmoscopy. Indirect ophthalmoscopy gives an image that is inverted vertically and horizontally, whereas direct ophthalmoscopy gives an erect image with no lateral inversion. The field of view is substantially greater with an indirect ophthalmoscope than with a direct ophthalmoscope. With the indirect ophthalmoscope, both the field of view and magnification vary according to the power of lens used.
  17. C One focal line is on the retina whilst the other is behind it
    In simple astigmatism, one focal line falls on the retina whilst the other is either in front or behind it. In simple hypermetropic astigmatism, the other focal line is behind it, as the eye does not have sufficient focussing power in that meridian.
    Compound astigmatism entails both focal lines falling either in front of or behind the retina. In mixed astigmatism, one focal line falls either side of the retina.
  18. C
    zoom view
    Toric transposition allows a base curve for the prescription to be specified. The steps are as follows:
    1. Ensure the cylinder in your prescription is the same sign as the base curve. If not, transpose it.
      1. In the question, both the cylinder (–1.00 DC) and the base curve (-4 D) are negative.
    2. Work out the numerator in the final toric formula (the sphere): from the sphere in your prescription, subtract the base curve.
      1. +2.00 – (–4) = +6.00
    3. Work out the denominator (the base curve, then the cylinder). The axis of the base curve is at 90° to the cylinder in your prescription. The final cylinder is obtained by adding the base curve power to the cylinder.
      1. The base curve is −4.00 D axis 90
      2. The cylinder is −4 + (−1) = −5.00 DC axis 180
      3. Put it all together:
        zoom view
        56
  19. D 30 D
    In indirect ophthalmoscopy the field of view increases with increasing power of examination lens, while the magnification decreases. The 15D lens is used for high magnification viewing of the posterior pole, while the 28D lens is often used in paediatrics where a wide field of view in a snap-shot is useful (it is also useful in patients with smaller pupils). 20D lenses are useful in general examination. 30D lenses are mostly used for patients with small pupils.
  20. A Virtual, erect, magnified, at infinity
    The Galilean telescope produces a virtual, erect, magnified image at infinity. The lenses within the telescope (a convex objective lens and a concave eyepiece lens) are separated by the difference between their focal lengths. The image formed can be shown in the ray diagram Figure 1.5.
    zoom view
    Figure 1.5:
  21. C The energy of a photon is directly proportional to its wavelength
    The energy of a photon is inversely proportional to its wavelength. The energy of a photon is defined by the equation:
    zoom view
    where E is the energy of a photon in joules, h is Planck's constant and v is the frequency of the photon in hertz.
    Therefore the energy of a photon is directly proportional to its frequency. Frequency is related to wavelength by the equation:
    zoom view
    where c is the speed of light in a vacuum and λ is the wavelength in metres, hence the energy of a photon is inversely proportional to its wavelength.57
    Another consequence of the above equation relating energy to frequency (and therefore wavelength) is that as some of the energy from the excitation light in fluorescence reactions is converted to thermal or chemical energy the light emitted will have a lower energy, and therefore a lower frequency and longer wavelength. This can be seen with fluorescein, which reacts to a cobalt blue excitation light (485–500 nm) and emits a yellow-green light (525–530 nm).
    Shorter wavelengths undergo increased scatter when compared with longer wavelengths. When caused by particles in a colloid or fine suspension this is known as the Tyndall effect, and causes the substance to appear blue (a common example is blue irides).
    The wavelength of light decreases when travelling into a material with a higher refractive index. As the absolute refractive index of a material is defined as the ratio of the speed of light in a vacuum to the speed of light in the material we can see that the speed of light decreases on entering any material with a higher refractive index. The frequency of a wave does not change on moving from one material to another, and therefore as the speed of light decreases the wavelength of light decreases by the same factor.
  22. A Destructive interference is used in anti-reflection coatings
    Anti-reflection coatings are classically designed so that their thickness is roughly a quarter of the wavelength of incident light. Therefore the reflected light from the film and the reflected light from the coated surface will be half a wavelength out of phase with each other, and hence produce destructive interference. Modern low-reflective coatings are often made up of several thin layers, but they still use the principle of destructive interference for their effect.
    Laser light generates interference more readily than conventional light as it is closer to true monochromatic light. This property is used in laser interferometry.
    The corneal stroma uses destructive interference in maintaining clarity. The collagen bundles are regularly spaced so that scattered light is eliminated by destructive interference.
    Two waves of equal amplitude need to be out of phase by half a cycle to cancel each other out by destructive interference.
  23. D Time spent outdoors as a child is a protective factor against myopia
    The prevalence of myopia worldwide varies considerably with ethnicity and geographical distribution, being particularly high in Southeast and East Asians compared to other ethnicities.
    Children are born relatively hypermetropic, which is overcome by their large amplitude of accommodation, and emmetropisation occurs as the eye grows. The prevalence of myopia is therefore higher in older children and adults than in young children.
    Many factors affect emmetropisation. Risk factors for myopia include genetic and environmental factors. A family history of myopia and high level of educational58 attainment are risk factors for myopia, whilst time spent outdoors is a protective factor. In Asia, low dose atropine has been used in children to slow the rate of progression of myopia.
 
Pathology
  1. D Renal cell carcinoma
    Acoustic neuromas (or vestibular schwannomas) are principally a feature of neurofibromatosis 2 (NF2), and nearly all patients with NF2 will develop them before the age of 30. However, they can also occur unilaterally in neurofibromatosis 1 (NF1).
    NF2 is associated with meningioma.
    The NF1 gene is on chromosome 17; NF2 is on chromosome 22. Both are autosomal dominant. The name neurofibromatosis is a misnomer in the case of NF2, as the principal tumour types are neuromas (schwannomas) and meningiomas.
    Phaeochromocytomas occur in NF1, as well as in Von Hippel–Lindau syndrome (VHL). Renal cell carcinomas occur in VHL, but are not associated with neurofibromatosis.
  2. D Sympathetic ophthalmia
    Dalen–Fuchs nodules are groups of epithelioid cells (activated macrophages) and lymphocytes between Bruch's membrane and the retinal pigment epithelium. They are a result of granulomatous inflammation at this site and can be seen in sympathetic ophthalmia, and also in other rare conditions including Vogt–Koyanagi–Harada (VKH) syndrome.
  3. A 12%
    Uveal tract melanomas may occur in the iris, ciliary body or choroid. Approximately 8% will occur in the iris, 12% in the ciliary body and 80% in the choroid. This is roughly equivalent to the relative tissue volumes of these structures.
  4. D There is characteristic muscle wasting, often involving facial and extraocular muscles
    Although myasthenia frequently causes weakness and fatigability in facial, bulbar and extraocular muscles it does not characteristically cause muscle wasting. The muscle weakness can be improved with an ice pack applied for 5 minutes. This is thought to be due to the inhibition of acetylcholinesterase at low temperatures. A Tensilon test may also be used. Cogan's lid twitch is seen when the patient returns59 to the primary position from downgaze. The upper lid initially overshoots, then ‘twitches’ downwards.
  5. C Hyperplasia
    Hyperplasia refers to an increase in the number of cells in a tissue. Hypertrophy refers to an increase in the size of individual cells. Both of these processes can cause a tissue to enlarge. Metaplasia refers to the change of differentiated cells to another cell type (this may be a less differentiated cell type). Dysplasia of cells refers to the proliferation of abnormal (often immature) cells in a tissue.
  6. A Amyloid appears as a homogenous blue material on haematoxylin and eosin H&E staining
    Amyloid is an insoluble fibrous protein and therefore takes up the pink eosin stain in an H&E stain, rather than the blue haematoxylin which binds to acidic structures such as nucleic acids. In lattice dystrophy amyloid deposits are seen in the corneal stroma. Systemic amyloidosis is significantly more common than localised amyloidosis. Amyloid exhibits ‘apple’ green birefringence when exposed to polarised light after staining with Congo red.
  7. C Outer plexiform layer
    Hard exudates are caused by localised hypoperfusion, leading to damage to the endothelium of deep capillaries and breakdown of the inner blood–retinal barrier. This causes leakage of plasma into the outer plexiform layer of the retina leading to what appears clinically as discrete, pale yellow deposits. Histologically, these stain pink on H&E, and contain foamy macrophages.
  8. D Unilateral
    Corneal arcus occurs most commonly as part of the normal ageing process, but is also seen in hyperlipidaemia. It is a fatty degeneration of the peripheral cornea that causes a circumferential opacity near the limbal margin. Lipid deposition occurs in the stroma. It is a bilateral condition: unilateral arcus is very rare and causes include ocular ischaemia and hypotony.
  9. A Accumulation of subretinal fluid occurs in all types of retinal detachment
    Retinal detachment is a separation of the neurosensory retina from the retinal pigment epithelium. There are three main types of retinal detachment (Table 1.7), all of which involve the accumulation of subretinal fluid. Rhegmatogenous is the commonest.60
    Table 1.7   Types of retinal detachment
    Type
    Pathophysiology
    Common causes
    Rhegmatogenous
    Liquefaction of vitreous, vitreoretinal traction, retinal tear or break
    Posterior vitreous detachment causing retinal tear or hole
    Dehiscence of the anterior retina at the ora serrata causing retinal dialysis (usually due to trauma)
    Exudative / Serous
    Compromised outer blood-retinal barrier
    Vascular, e.g. wet age-related macular degeneration
    Inflammatory, e.g. posterior scleritis Neoplastic, e.g. choroidal melanoma
    Tractional
    Contracting proliferative retinal membranes, abnormal vitreoretinal adhesions
    Proliferative diabetic retinopathy Proliferative vitreoretinopathy Retinopathy of prematurity
  10. C Raised intraocular pressure is common, due to inflammation
    Proliferative vitreoretinopathy is a common cause of failure following retinal detachment surgery and is more common with large breaks, multiple operations and duration of retinal detachment. It may also occur in untreated retinal detachment. Following the break in the retina, retinal pigment epithelial cells migrate into the subretinal space and vitreous. The blood–retinal barrier is broken and inflammatory cells are recruited. Fibrosis and collagen production occur, contributing to the formation of retinal membranes. Contraction of these can lead to retinal detachment and, if untreated, ocular hypotony (not raised intraocular pressure) and ultimately phthisis bulbi.
  11. B Regeneration of the epithelium and Bowman's layer
    Bowman's layer does not regenerate. The phases of healing of a full-thickness corneal laceration are listed in Table 1.8.
    Table 1.8   Phases of healing of a full-thickness corneal laceration
    Time
    Phase of healing
    Minutes
    Retraction of Descemet's and stromal collagen
    Fibrin plug formation
    Stromal oedema
    Invasion of leukocytes from aqueous and limbal vessels
    Hours
    Transformation of leukocytes and of stromal keratocytes into fibroblasts
    Collagen formation
    Start of epithelial regeneration
    Day(s)
    Endothelial sliding, filling in gaps in endothelium and Descemet's
    Weeks
    Replacement of stroma and Bowman's with scar tissue61
 
Pharmacology
  1. A Brief blockade of synaptic transmission
    Botulinum toxin A is a potent neurotoxin which inhibits the release of acetylcholine at the neuromuscular junction. It achieves this by cleaving SNARE proteins, which are required for fusion of the neurosecretory vesicles with the plasma membrane, thus blocking the exocytosis of acetylcholine. It causes long-lasting blockade of synaptic transmission. Botulinum toxin poisoning is characterised by progressive parasympathetic and motor paralysis. Botulinum toxin A is used therapeutically by local injection into muscles in the treatment of blepharospasm and strabismus, and to induce ptosis.
  2. D Lipid-soluble drugs can pass the blood–retinal barrier
    Drugs absorbed from the gastrointestinal tract pass through the portal circulation before reaching the systemic circulation and may be metabolised by the liver in a process known as first-pass metabolism. Although eye drops often have few systemic side effects, this is not always the case as any drug absorbed systemically via the nasopharyngeal mucosa avoids first pass metabolism.
    The metabolism of drugs can be described in terms of first- or zero-order kinetics. First-order kinetics are linear where the rate is proportional to the concentration of drug. In zero-order kinetics, the metabolism of the drug is independent of the concentration (non-linear).
    Drug distribution depends on several factors including protein binding and lipid solubility. Highly protein-bound drugs tend to remain in the plasma compartment. In general, lipid solubility increases drug absorption and distribution, for example by allowing diffusion across cell membranes and the blood–retinal barrier.
  3. A Bone marrow suppression
    Ciclosporin, along with tacrolimus, is a calcineurin inhibitor whose immunosuppressant effects are via action on T lymphocytes. It inhibits the transcription of IL-2 and other cytokines. Side effects include nephrotoxicity, tremor, hirsutism and gingival hypertrophy.
  4. B Hydroxyamphetamine 1%
    The working diagnosis in this patient is Horner's syndrome secondary to a Pancoast tumour in the pulmonary apex. This is a preganglionic lesion affecting the second-order neurone.
    Failure to dilate to cocaine is diagnostic of Horner's syndrome regardless of the level of the lesion. Cocaine inhibits the reuptake of noradrenaline at sympathetic synapses, so has no effect if no noradrenaline is secreted. In this patient, the right pupil should fail to dilate to cocaine.62
    Failure to dilate to hydroxyamphetamine indicates a postganglionic lesion, i.e. affecting the third-order neurone. This is because it causes release of noradrenaline from postganglionic nerve terminals which can only occur with an intact 3rd order neurone. In this patient, both pupils should dilate.
    These agents are learnt for exams but are not usually available in a modern eye clinic. Apraclonidine and dilute phenylephrine (1%) have been used as alternatives (Table 1.9), although the positive reaction in Horner's is a dilatation, rather than a failure to dilate. Note that denervation hypersensitivity to apraclonidine is due to upregulation of iris α1 receptors and may take several days to develop. Thus a negative apraclonidine test in the acute setting does not exclude Horner's syndrome.
    Pilocarpine constricts rather than dilates the pupil so is not helpful.
    Table 1.9   Pharmacological agents used in the diagnosis of Horner's syndrome
    Positive result
    Agent
    Level of lesion
    Mechanism
    Fails to dilate
    Cocaine 4%
    All Horner's
    Inhibits NA reuptake at sympathetic synapses, so no effect if no NA secreted
    Hydroxyamphetamine 1%
    3rd order lesion
    Stimulates release of NA from postganglionic nerve terminals
    Dilates, i.e. reversal of anisocoria
    Apraclonidine 0.5% or 1%
    All Horner's
    α2 agonist with weak α1 activity Denervation hypersensitivity
    Phenylephrine 1%
    3rd order lesion
    α1 agonist
    Denervation hypersensitivity
    NA, noradrenaline.
  5. A α2 agonist
    Brimonidine is an α2 adrenoreceptor agonist which lowers aqueous production as well as increasing uveoscleral outflow.
    The four possible answers listed are the principal classes of topical medication used in the treatment of open angle glaucoma and ocular hypertension. The pros and cons of each drop must be weighed up for an individual patient when making treatment decisions. When compared to the other classes, α2 agonists such as brimonidine carry more risk of allergy (approximately 10%). However, they are considered safer than the others in pregnancy, and they are not contraindicated in asthma (unlike beta-blockers).
  6. A Hyperkalaemia
    Acetazolamide is a carbonic anhydrase inhibitor used as a systemic intraocular pressure-lowering agent.
    Patients commonly experience paraesthesia of the extremities. Additionally, serious side effects of acetazolamide include hypokalaemia and potassium depletion,63 metabolic acidosis, renal stones, depression; more rarely, blood disorders and Stevens–Johnson syndrome.
    Due to this side effect profile, long term use of acetazolamide is not generally recommended. However, it is highly valuable in gaining control of intraocular pressure in certain situations, for example in the management of acute angle-closure glaucoma.
  7. D Thromboxane A2 causes vasoconstriction
    Eicosanoids are an important group of signalling molecules and inflammatory mediators that include prostaglandins and thromboxanes (together termed prostanoids), and leukotrienes. They can be generated as required in response to stimuli. The inflammatory response always involves prostanoid generation.
    The precise stimuli vary depending on cell type and include cell damage, thrombin (platelets), complement-derived C5a (neutrophils) and bradykinin (fibroblasts). They are derived from arachidonic acid, which is liberated from cell membrane phospholipids in a rate-limiting step catalysed primarily by phospholipase A2. The enzymes cyclooxygenase (COX-1 and COX-2 isoforms) and lipoxygenase then catalyse the production of different eicosanoids from free arachidonic acid.
    Nonsteroidal anti-inflammatory drugs (NSAIDs) reduce inflammation by inhibiting cyclooxygenase and therefore the biosynthesis of prostaglandins, but subsequently have no effect on their actions on target tissue. Prostaglandins exert their effects by acting on G-protein coupled receptors.
    Thromboxane A2 is an eicosanoid which causes vasoconstriction and platelet aggregation.
  8. C Midazolam
    Benzodiazepines are used for sedation, premedication and induction of anaesthesia, and as anxiolytics, anticonvulsants and muscle relaxants. They act on the GABAA receptor in the central nervous system.
    The half-lives of different benzodiazepines, outlined in Table 1.10, are a key factor in determining their use. Whilst the exact half-lives need not be memorised (and vary between published sources) it is important to know, for example, that midazolam is short acting making it suitable for sedation for short procedures and induction.
    Table 1.10   Half-lives of different benzodiazepines
    Benzodiazepine
    Half-life (hours)
    Midazolam
    1-3
    Temazepam
    6-8
    Lorazepam
    12
    Diazepam
    12-2464
 
Physiology
  1. C Increased vascular permeability
    Histamine exerts its physiological effects by acting on histamine receptors, of which there are three types: H1, H2 and H3. They are all G-protein coupled receptors. Histamine is an inflammatory mediator and its effects in type I hypersensitivity reactions are via the H1 receptor. Its gastric effects are H2-mediated. H3 receptors are expressed mainly in the central nervous system and affect the release of various neurotransmitters.
    Histamine is released during, but does not cause, mast cell degranulation. Important actions of histamine via the H1 and H2 histamine receptors are shown in Table 1.11.
    Table 1.11   Histamine receptors
    Receptor
    Intracellular effects
    Physiological effects
    H1 receptor
    Activation of phospholipase C Increased intracellular calcium concentration
    Increased vascular permeability
    Vasodilatation
    Central nervous system depressant
    H2 receptor
    Activation of adenylate cyclase and increase in cAMP
    Increased gastric acid production Increased cardiac stroke volume
  2. A Chromaticity depends on hue, saturation and luminance
    Most of the light we see is reflected from objects, and the nature of the reflected light determines the object's colour. ‘Colour’ essentially refers to chromaticity and depends on:
    • Hue
    • Saturation
    • Luminance
    Hue is not independent of luminance (which in this context could also be referred to as luminosity or brightness): with increasing brightness, eventually all hues appear yellow-white (the Bezold–Brücke effect), whilst with very low luminosity all hues appear achromatic.
    The perceived colours of objects around us are a product of both the light source and the surface properties of the objects, which determine the composition of the reflected light.
    The maximal spectral sensitivity of green cones is 535–550 nm.
  3. C Increased cone sensitivity accounts for the early component of the dark adaptation curve
    Dark adaptation is the process by which the retina increases in sensitivity to light in response to decreasing background illumination.65
    The dark adaptation curve plots the light intensity required to perceive a spot of light versus time. It is bipartite, with an initial rapid (5–10 minutes) increase in cone sensitivity and then a slower period (15–30 minutes) where rods reach their maximum sensitivity. Thus dark adaptation is complete in 30 minutes in normal subjects.
    Fully dark adapted rods allow perception of a spot of light at least 100 times dimmer than cones.
    Light adaptation occurs more rapidly than dark adaptation.
  4. B Insulin promotes the movement of potassium from the intracellular to the extracellular compartment
    Insulin promotes uptake of potassium by cells.
    Potassium is the chief intracellular cation, with an intracellular fluid concentration of approximately 135–150 mmol/L (versus 3.5–5 mmol/L in extracellular fluid, aqueous and plasma). This gradient is largely maintained by the Na+/K+-ATPase pump. Over 90% of total body potassium is intracellular.
    Daily potassium requirements are approximately 1 mmol/kg/day. Renal handling of potassium is crucial in maintaining potassium homeostasis. Potassium is reabsorbed at the proximal convoluted tubule and then secreted back into the urine by the distal convoluted tubule (promoted by aldosterone). Renal potassium excretion is also influenced by antidiuretic hormone (ADH), tubular flow rate and acid–base balance.
  5. B Atrial contraction is responsible for 80% of ventricular filling
    The cardiac cycle is divided into five phases (Table 1.12).
    Although contraction of the two sides of the heart occurs during the same phases of the cardiac cycle, it is slightly asynchronous:
    • Right atrial contraction begins before left
    • Left ventricular contraction begins before right
    Rapid passive filling of the ventricles occurs in early diastole. Atrial contraction is responsible for only 30% of ventricular filling.
    On an electrocardiography (ECG) trace of the cardiac cycle, the P wave corresponds to atrial depolarisation.
    Table 1.12   Phases of the cardiac cycle
    1
    Atrial systole
    2
    Isometric ventricular contraction
    Ventricular systole
    3
    Ventricular contraction
    4
    Isometric ventricular relaxation
    Diastole
    5
    Ventricular filling
  6. 66B Somatostatin promotes the release of growth hormone from the anterior pituitary
    The hypothalamus secretes releasing or release-inhibiting hormones which act on the pituitary gland. These are often referred to as ‘factors', e.g. corticotrophin-releasing factor (CRF), thyrotrophin-releasing factor (TRF) and growth hormone-release inhibiting factor (better known as somatostatin).
    The actions of growth hormone, secreted by the anterior pituitary, include increased skeletal growth, protein synthesis, gluconeogenesis and lipolysis. The effects of over-secretion depend on whether it occurs before puberty (gigantism) or after puberty (acromegaly).
    The secretion of hormones from the anterior pituitary, including growth hormone and adrenocorticotropic hormone (ACTH), is pulsatile.
  7. B Oncotic pressure
    Oncotic pressure is generated by plasma proteins and works against aqueous humour production. Recall that the aqueous is virtually free of protein in the healthy state. Production of aqueous is maintained by active secretion in the double-layered ciliary epithelium, passive diffusion of ions down their concentration and charge gradients and ultrafiltration of water and solutes secondary to the capillary hydrostatic pressure. Of these processes active secretion is thought to account for 80–90% of aqueous humour production.
  8. D 4
    The primary visual cortex (V1), like all cerebral cortical areas, is divided into 6 basic layers. The neurones of the optic radiations travel from the lateral geniculate nucleus to layer 4 of the primary visual cortex and synapse there. Layer 4 of the primary visual cortex is expanded into distinct sublayers with a central white line known as the stria of Gennari. This represents the myelinated fibres of the optic radiations. It is because of this appearance of distinct sublayers that the primary visual cortex is sometimes referred to as the striate cortex.
  9. B Cells in layer 5 project principally to the secondary visual cortex
    Cells in layer 5 project to the superior colliculus. Input to the secondary visual cortex is by layers 2 and 3. Layer 6 provides feed-forward innervation to the lateral geniculate nucleus. The superior lip of the calcarine sulcus receives input from the superior retina (and therefore the inferior visual field), and vice versa for the inferior lip of the sulcus. There are alternating ocular dominance columns in layer 4 C, each responsive to inputs from a single eye. Thus neurones corresponding to each eye are segregated as far as the primary visual cortex.67
  10. B Circadian rhythm
    The suprachiasmatic nucleus receives input from the retina via the retinohypothalamic tract and generates the circadian rhythm cycle via the action of pacemaker cells. These influence the production of melatonin in the pineal gland, which in turn allows the organism to adjust physiologically to the light–dark cycle. As the light–dark cycle changes during the course of the year the suprachiasmatic nucleus permits the organisation of physiological functions on both a diurnal and a seasonal basis.
    A summary of subcortical structures related to vision is given in Table 3.9 (page 230).
  11. D Keratan sulphate
    All of these glycosaminoglycans are found in the cornea, but keratan sulphate is thought to be crucial for maintaining corneal clarity. It binds to collagen arrays and is thought to maintain interfibrillar distance.
  12. D The magnitude of an action potential determines the quantity of neurotransmitter released
    Action potentials are an ‘all-or-nothing’ response, in that when they are initiated they will always obtain the same magnitude. Information is instead transmitted by the number of action potentials and their frequency.
    The cell membranes of neurones usually have a negatively charged resting membrane potential, which can be depolarised by various stimuli (for example the opening of ligand-gated ion channels on their dendrites). The membranes of neurones contain voltage-gated sodium channels which will allow an influx of Na+ ions when a threshold voltage is reached. This further depolarises the cell, generating a feedback loop and triggering an action potential. Once the cell is depolarised the voltage-gated sodium channels close and voltage-gated potassium channels open. This causes an outward flow of K+ ions, repolarising the cell membrane. The density of voltage-gated sodium channels is highest at the initial segment of the axon (the axon hillock for motor neurones and interneurones, and the first node of Ranvier for myelinated sensory neurones), and so this is where action potentials are initiated.
  13. C Vergence movements
    Disconjugate movements are when both eyes move in opposite directions, and are seen in vergence movements where the eyes either converge or diverge. Other eye movements are conjugate, i.e. both eyes move in the same direction. Key eye movements are listed below in Table 1.1368.
    Table 1.13   Key eye movements
    Eye movement
    Key features
    Saccades
    Rapid ballistic movement to change point of fixation
    Voluntary or reflexive
    May be small (e.g. when reading) or large (e.g. looking up from your desk)
    Maximum velocity ~900°/second
    Saccades
    Slow tracking movement to keep a target on the fovea
    Voluntary movement (although requires a moving stimulus)
    Maximum velocity ~100°/second
    Vergence
    Disconjugate movement (convergence or divergence)
    Generally used to change between near and distance focus points
    Uses retinal disparity to calculate movement required
    Part of the accommodation response
    Slower than smooth pursuit movements
    Vestibulo-ocular
    Stabilise the eye relative to the external world
    Rapid movements to compensate for changes in head position
    Reflex movement
    Independent of visual input
  14. C Right inferior oblique and left superior oblique
    Yoked muscles are pairs of extraocular muscles designed to bring about a co-ordinated direction of gaze. Each medial rectus is paired with the opposite lateral rectus, each superior oblique is paired with the opposite inferior rectus and each inferior oblique is paired with the opposite superior rectus.
  15. A Ciliary muscle relaxation
    The ciliary muscle contracts during accommodation, allowing the increase in lens curvature and therefore refractive power. The eyes converge to focus on the near target and the pupil constricts to restrict the passage of light to the central lens.
  16. A Depolarisation of the hair cell in the cochlea occurs due to the influx of sodium
    Unlike most other excitable tissues, depolarisation in the hair cell occurs due to an influx of potassium (not sodium) from the endolymph.
    Sound waves cause vibrations of the tympanic membrane, which are transmitted and magnified across the middle ear by the ossicles to the inner ear. Vibrations in the endolymph cause vibrations in the basilar membrane of the cochlea which is where the organ of Corti is located. The organ of Corti transduces the vibrations into action potentials in the auditory nerve.
    Sound is audible to humans between 20 and 20,000 Hz, with the greatest sensitivity being in the 1000–4000 Hz range. Different pitches stimulate hair cells at different locations in the organ of Corti. Higher frequencies are detected by hair cells near the base of the cochlea, and low tones near the apex.69
 
Statistics and evidence-based medicine
  1. C 70%
    Sensitivity is a statistical value which measures the ability of a test to correctly assign a positive outcome to a patient. It is defined as the probability of the test being positive when the disease is present. It can be calculated by the formula below:
    zoom view
    where x is the number of true positives (tested positive and found to be positive) and y is the number of false negatives (tested negative and found to be positive).
    In this case:
    zoom view
  2. A Failing to reject the null hypothesis when it is false
    A type II error is when the study fails to reject the null hypothesis even though it is false, i.e. failing to detect a real difference. Conversely a type I error is where a null hypothesis is rejected when it is true, i.e. finding a difference when there is none.70
 
Answers: CRQs
 
Anatomy
  1. Answer
    1. The diagram shows a coronal section through the cavernous sinus, midway along the body of the sphenoid bone. Inferior to the cavernous sinus are the sphenoid air spaces (unlabelled), and above it the optic chiasm (not pictured). The labelled structures are:
      1. Maxillary division of trigeminal nerve (V2)
      2. Ophthalmic division of trigeminal nerve (V1)
      3. Abducens nerve (VI)
      4. Internal carotid artery
      5. Trochlear nerve (IV)
      6. Oculomotor nerve (III)
      7. Pituitary gland (hypophysis cerebri)
    2. Two conditions which typically affect the cavernous sinus are:
      • Carotid–cavernous fistula (arteriovenous fistula also acceptable)
      • Cavernous sinus thrombosis
      Feedback
      A carotid–cavernous fistula is an abnormal arteriovenous communication between the internal carotid artery and the cavernous sinus, resulting in arterialisation of the venous flow with raised venous pressure and venous stasis. Carotid–cavernous fistulae may be traumatic, spontaneous (usually elderly patients) or congenital. They may be direct, high-flow shunts, presenting with a red eye with pulsatile proptosis; or indirect, low-flow shunts where the arteriovenous communication is at the level of smaller branches of the carotid arteries, with more subtle clinical features.
      The superior and inferior ophthalmic veins and the facial vein all communicate with the cavernous sinus. Thus infection of the facial skin, orbit or paranasal sinuses may lead to the spread of septic emboli and resulting cavernous sinus thrombosis. Noninfectious cavernous thrombosis also occurs (often with a more subtle presentation), with risk factors including prothrombotic states such as malignancy and oral contraceptive use. The clinical features of cavernous sinus thrombosis depend on the structures affected and are caused by venous congestion, cranial nerve involvement or any underlying infection.
    3. The abducens nerve (VI)71
      Feedback
      The abducens nerve (VI) is the most vulnerable due to its intraluminal course through the cavernous sinus. The other cranial nerves pass along the wall of the sinus which offers a degree of protection; III, IV and V involvement is therefore less common.
 
Biochemistry and cell biology
  1. Answer
    1. Synaptic vesicles
      Feedback
      Neurotransmitter molecules at the synaptic terminal are stored in synaptic vesicles (also known as presynaptic vesicles) prior to release.
    2. Noradrenaline (norepinephrine)
      Feedback
      Postganglionic sympathetic fibres release noradrenaline (also called norepinephrine) as their principal neurotransmitter, except the neurones innervating the adrenal medulla and the post-ganglionic sympathetic neurones of sweat glands.
    3. Tyrosine
      Feedback
      Tyrosine (or 4-hydroxyphenylalanine) is converted to L-DOPA by the enzyme tyrosine hydroxylase. L-DOPA is then converted to dopamine by DOPA decarboxylase. Dopamine β hydroxylase converts dopamine into noradrenaline.
    4. Dopamine and adrenaline (epinephrine) (1 mark for each)
      Feedback
      As described above, dopamine is the metabolic precursor to noradrenaline and is an important neurotransmitter in several systems in the central nervous system. Adrenaline (also called epinephrine) is synthesised from noradrenaline by the action of phenylethanolamine N-methyltransferase (PNMT), and acts as a neurotransmitter as well as a circulating hormone.
    5. Exocytosis
      Feedback
      Neurotransmitters are released by exocytosis. The synaptic vesicles containing neurotransmitter molecules fuse with the cell membrane at the synaptic terminal, releasing neurotransmitter into the synaptic cleft.72
    6. Metabotropic (G protein-coupled) receptors and ionotropic (ligand-gated ion channel) receptors. G protein-coupled receptors are responsible for transmission at this synapse (1 mark for each type of receptor, mark given if either nomenclature used, and a further mark for correctly identifying the type at this synapse)
      Feedback
      The two classes of receptors found at synapses are G protein-coupled receptors, which exert their influence by activating an intracellular G protein which in turn modulates other intracellular signalling molecules; and ligand-gated ion channel receptors, which incorporate an ion channel which will open or close depending on the binding of different ligands (e.g. a neurotransmitter). Non-G protein-coupled metabotropic receptors can be found elsewhere as receptors for hormones, cytokines and growth factors (e.g. receptor tyrosine kinases).
      The principal post-synaptic receptors at this synapse are α1 adrenoreceptors, which like all adrenoreceptors are G protein-coupled.
    7. Acetylcholine
      Feedback
      Preganglionic sympathetic nerves, including those entering the superior cervical ganglion, release acetylcholine as their principal neurotransmitter. This acts on nicotinic acetylcholine receptors on the postganglionic sympathetic nerves.
 
Genetics
  1. Answer
    • (a)
      zoom view
      73
      (5 marks for accurate pedigree, indicating proband, affected individuals, gender, deceased individual, pregnancy)
      Feedback
      The standardised rules and symbols for pedigree drawing include the following:
      A square denotes a male individual; a circle denotes a female; and a diamond an individual of unknown sex.
      It is good practice to write their age or year of birth underneath. Do not write the age in the symbol; this is used for multiple individuals, e.g. a circle with number 5 in it indicates 5 unaffected females. Siblings should appear in age order with the eldest on the left.
      The symbol is coloured in to denote affected individuals.
      A marriage or partnership is indicated by a horizontal line between two symbols. A double horizontal line indicates a consanguinous marriage.
      An arrow denotes the proband.
      A pregnancy is indicated by a P within the symbol.
      Deceased individuals are indicated by a diagonal line through their symbol.
    • Autosomal dominant
      Feedback
      The features shown here that are characteristic of autosomal dominant inheritance are: affected individuals in each generation, both males and females affected, and multiple forms of transmission of the affected gene (male to female, female to female).
    • 50%, the risk is the same for a boy or a girl
      Feedback
      In autosomal dominant conditions, there is a 50% risk of each child being affected, and the risk is not affected by the sex of the child. This assumes that the affected parent is a heterozygote for the gene in question, and the partner is unaffected. In this example, we know that this is the case as the proband's mother and husband are unaffected.
      The risk is affected by the sex of the child in X-linked conditions. Most X-linked conditions are X-linked recessive. In these conditions, males are almost exclusively affected (the ‘almost’ is due to the phenomenon of X-inactivation). As a father only passes on the Y chromosome to his son, the gene is passed on to males via the mother. Therefore, on a pedigree only female to male transmission is seen. All of an affected father's daughters will be carriers of the gene.
    • Aniridia
      Feedback
      Aniridia is characterised by a spectrum of iris hypoplasia, ranging from transillumination defects in the iris to near-complete absence of the iris. This is often74 accompanied by foveal hypoplasia (+/– optic nerve hypoplasia) which tends to cause pendular nystagmus and poor vision.
      In aniridia, as well as in iridocorneal dysgenesis conditions such as Peter's and Rieger's anomalies, glaucoma occurs in over 50% of individuals. This is due to abnormalities in the trabecular meshwork. Other complications of aniridia include cataracts, and keratopathy secondary to corneal epithelial stem cell failure.
    • PAX-6
      Feedback
      Aniridia is caused by mutations or deletions affecting the PAX-6 gene on chromosome 11. The commonest mutations result in autosomal dominant inheritance (2/3 of cases) and no extraocular features. Sporadic cases (1/3) may have extraocular features. For example, a partial deletion in the short arm of chromosome 11 may affect the adjacent WT1 tumour suppressor gene. This can lead to WAGR syndrome, consisting of Wilm's tumour (nephroblastoma), Aniridia, Genitourinary malformations and mental Retardation. Aniridia is also a feature of the rare Gillespie syndrome (including cerebellar ataxia), which can also be recessive.
 
Investigations and imaging
  1. Answer
    1. Macular OCT (optical coherence tomography) scan
    2. A broadband, low coherence light source is directed at the desired target (1 mark). A beam splitter is used to simultaneously direct the same light source at a reference mirror (1 mark). The reflected light from both the desired target and the reference mirror are directed onto a detector, and the interference pattern is analysed using low coherence interferometry to construct an image of the desired target (1 mark)
    3. Any two of: it requires transparent media; it requires patient co-operation; it requires at least moderate dilatation; it is susceptible to motion artefact (1 mark for each, up to 2 marks)
    4. Cystoid macular oedema
      Feedback
      There is an abnormal accumulation of fluid in cystoid spaces within the inner retina. The most common site of fluid accumulation is the outer plexiform layer, most probably because this is the watershed area between the choroidal and retinal circulations.
    5. Any two of: postoperative inflammation, especially after cataract extraction (Irvine-Gass syndrome); uveitis; retinitis (e.g. CMV retinitis); diabetic maculopathy;75 retinal vein occlusions; vitreomacular traction; retinitis pigmentosa; topical adrenaline or prostaglandin E2 therapy; nicotinic acid therapy; niacin therapy
      Feedback
      Cystoid macular oedema is a shared pathological response to a number of inflammatory and retinal vascular processes. It can lead to blurred and/or distorted central vision, and remains a significant cause of reduced vision after cataract surgery. There are a number of rarer causes of cystoid macular oedema, including several iatrogenic causes. There is sometimes a cystoid component in diabetic maculopathy. This is more common in diffuse and chronic diabetic macular oedema.
    6. Fluorescein angiography
      Feedback
      Fluorescein angiography is particularly useful for investigating retinal vascular disorders.
  2. Answer
    1. Hoffer Q is likely to be more accurate for the left eye, and the axial length allows you to determine this (1 mark for Hoffer Q; 1 mark for axial length)
      Feedback
      The Hoffer Q formula has been demonstrated to be more accurate than the SRK/T formula for eyes with an axial length below 21.5 mm. Conversely the SRK/T formula was demonstrated to have better accuracy for eyes with an axial length of 27 mm or greater. As the axial length in this patient's left eye is 20.93 mm, it would be advisable to use the Hoffer Q formula to predict the correct intraocular lens implant.
    2. Axial length and average keratometry (corneal curvature or power) (1 mark for each; either nomenclature may be used for the average keratometry)
      Feedback
      Both the SRK/T and Hoffer Q formulas rely on accurate measurement of the axial length and the average keratometry. The Holladay 1 formula also relies on these two measurements. The Haigis formula additionally uses pre-operative anterior chamber depth, while the Holladay 2 formula uses seven measurements.
    3. Axial length, caused by corneal indentation with contact ultrasound biometry (1 mark for axial length; 1 mark for corneal indentation)
      Feedback
      Contact ultrasound biometry always produces some corneal indentation and therefore the axial lengths recorded are shorter than those measured by immersion or optical biometry.
    4. It is not always possible to measure axial length using optical biometry76
      Feedback
      In approximately 10% of eyes it is not possible to measure the axial length using optical biometry. This is particularly a problem with dense posterior subcapsular cataracts. In these cases it is necessary to measure the axial length using ultrasound.
    5. 62 degrees or 242 degrees (1 mark for either)
      Feedback
      The steep axis is indicated by the higher K value (in this case K2: 45.79 D at 62 degrees). Any astigmatism caused by a corneal incision at this point will reduce the steepness of this axis and therefore reduce the total astigmatism of the eye. This intraocular lens printout also has the effective negative cylinder calculated below the K values, and the steep axis will be at 90 degrees to the axis of the negative cylinder.
    6. 118.5
      Feedback
      As in the original SRK formula, the A-constant in the SRK/T formula has a 1:1 relationship with the intraocular lens (IOL) power. Therefore if two lenses have the same predicted outcome the difference between the IOL powers will be the difference between the A-constants. The biometry for the right eye gives a predicted outcome of −0.09 for a 29 dioptre SN60WF IOL.
      Therefore for a lens giving the same outcome with a 28.5 dioptre IOL the A-constant must be 0.5 dioptres smaller than that of the SN60WF, therefore 118.5.
  3. Answer
    1. Indocyanine green (ICG) angiography
      Feedback
      Image B is a slide from indocyanine green (ICG) angiography. The contrast with the fundus fluorescein angiography (FFA) image, A, is stark: rather than the diffuse background fluorescence of FFA it is possible to see the individual choroidal vessels in addition to the vessels of the retinal circulation.
    2. Both the excitation source and fluorescence for investigation B are in the near-infrared range, with the fluorescence having a longer wavelength than the excitation source (1 mark each for excitation source and fluorescence, with either a statement of near-infrared or a reasonable wavelength or range given)
      Feedback
      In vivo, indocyanine green (ICG) absorbs near-infrared light between approximately 790–805 nm. Its emission spectrum is also near-infrared in the range of 770–880 nm, with a peak emission of 835 nm. This is an important factor in the relative advantages of ICG over fluorescein angiography (see below).77
    3. Both the excitation and fluorescence lights of fluorescein angiography are absorbed and scattered by pigments in the retina and retinal pigment epithelium (including macular xanthophyll), limiting the visualisation of structures deep to these tissues. Also fluorescein molecules rapidly extravasate from the choroidal circulation, giving rise to a diffuse background fluorescence (1 mark each for: scattering and absorption of excitation and fluorescence light; rapid extravasation from choroidal circulation)
      Feedback
      The relatively short wavelengths of the excitation and fluorescence lights in fluorescein angiography result in a greater degree of scatter and absorption by the tissues of the fundus than the near-infrared rays of indocyanine green (ICG) angiography. Indeed macular xanthophyll (mainly found in the inner and outer plexiform layers) is thought to play a physiological role in limiting damage to photoreceptors from high-energy short wavelength light. Also as fluorescein extravasates more rapidly than ICG it quickly fills the area of the choroid with a low intensity haze that further limits the view of the choroidal circulation.
    4. Any three of:
      Better visualisation through media opacities
      Better visualisation of vessels behind haemorrhages
      Better tolerance in photophobic patients
      More accurate measurement of the size of occult choroidal neovascular membranes (CNVM)
      (1 mark for each up to three marks)
      Feedback
      As indocyanine green (ICG) utilises light in the near-infrared spectrum the scattering from media opacities is much lower than with the shorter wavelengths used in fluorescein angiography. For a similar reason it is possible to visualise large blood vessels hidden deep to haemorrhages in the retina. As near-infrared light is barely perceptible to the human eye it is better tolerated by photophobic patients than the cobalt blue excitation light of fluorescein angiography. It is difficult to accurately measure the size of occult CNVMs with fluorescein angiography as they may be over-estimated due to leakage into neurosensory or pigment epithelial detachments, or under-estimated due to blocked fluorescence by blood or exudate.
    5. Because indocyanine green (ICG) has a higher degree of binding to plasma proteins than fluorescein
      Feedback
      ICG is 98% bound to plasma proteins, with around 80% being bound to globulins (especially alpha-1-lipoproteins). Fluorescein, by comparison, is 80% protein bound in the circulation, leaving more free molecules to extravasate. This difference is largely due to the ampiphilic nature of ICG (both hydrophilic and lipophilic) compared to the primarily hydrophilic nature of fluorescein, resulting in ICG binding to lipoproteins and phospholipids in addition to the proteins fluorescein has an affinity for.78
    6. Hepatobiliary excretion
      Feedback
      Indocyanine green (ICG) is excreted entirely by the hepatobiliary system. Hepatic parenchymal cells take up ICG and secrete it unchanged into the bile. This typically results in discolouration of the stool for several days. Persistence of the dye in the retinal and choroidal circulation for more than 30 minutes in the late phase should raise the suspicion of reduced hepatic function.
  4. Answer
    1. An X-ray tube rotates around the patient, taking hundreds of X-rays from different angles, which are detected by detector plates and processed by computer to produce the CT images (1 mark for describing a rotating X-ray tube and detector; 1 mark for describing processing of images by computer)
      Feedback
      It is important to understand that each axial segment in a CT scan is compiled from a number of X-ray images taken at different angles which only make a useful image when put together. CT scanners use rows of small, electronic detectors rather than conventional film, so that the X-ray data can be easily compiled by the computer. CTs are acquired in the axial plane, however the data can subsequently be reconstructed into any plane. 3D reconstructions are also possible.
    2. Opacification of the right maxillary sinus and soft tissue swelling/stranding in the right peri-orbital/facial area (1 mark for each abnormality)
      Feedback
      Sinuses are usually black on a CT scan (as they are air-filled), and subtotal opacification as seen in this image is indicative of a more radio-dense material in the sinus. This could be serous fluid, pus, haemorrhage or solid material, and the density of the material, which can be assessed on CT, can help to differentiate between these. The right peri-orbital/facial soft tissues are significantly swollen compared with the left and there is a haziness of the subcutaneous fat (stranding) in keeping with acute inflammation.
    3. Right proptosis
      Feedback
      When looking at CT or MRI images you should always compare the structures on each side for asymmetry. In this scan the right globe is significantly further forwards in the orbit than the left. It is also possible to see more of the soft tissue swelling, and further opacification of the paranasal sinuses (right ethmoid sinus).
    4. Orbital cellulitis (secondary to sinusitis) (1 mark for orbital cellulitis)
      Feedback
      These features are most in keeping with an orbital cellulitis secondary to sinusitis.79 Bacterial spread from paranasal sinuses is the most common aetiology for orbital cellulitis.
    5. Right relative afferent pupillary defect (RAPD), which would indicate compromise of right optic nerve function (1 mark for right RAPD; 1 mark for significance)
      Feedback
      It is important to look for an RAPD whenever orbital cellulitis is suspected as this will help to differentiate blurred vision secondary to surface factors from blurred vision secondary to optic nerve compromise.
    6. Any two of:
      • Meningitis
      • Cerebritis
      • Cavernous sinus thrombosis
      • Intracranial abscess
      • (1 mark for each correct answer; only two answers should be provided)
      Feedback
      Intracranial spread of infection is a potentially serious complication of orbital cellulitis. It can involve the meninges (meningitis) or brain parenchyma (cerebritis). Abscesses can form within the brain parenchyma or in the extra-axial space (e.g. subdural empyema). The inflammatory process can also cause cavernous sinus thrombosis. Complications are more likely if presentation or treatment is delayed. Thankfully with appropriate treatment these complications are rare.
 
Microbiology
  1. Answer
    1. Fungal hyphae
      Feedback
      This is a slide showing fungal hyphae, which are branching filamentous structures that grow from their tips to facilitate the spread of the organism.
    2. Sabouraud agar
      Feedback
      Sabouraud agar is formulated to support the growth of a range of fungi. It has a relatively acidic pH to inhibit the growth of bacteria.
    3. Culturing of fungi can be slower than clinical progress80
      Feedback
      Fungi are slow-growing in culture and it can take up to 3 weeks to culture and identify the organism. In this time the patient's clinical condition will have progressed significantly.
    4. Acridine orange and calcofluor white (1 mark for each)
      Feedback
      Acridine orange and calcofluor white will both stain fungi and cause them to fluoresce under the appropriate lighting conditions. This can assist with the rapid identification of pathogens. A number of other stains may be used which do not require a fluorescent microscope, including Periodic Acid Schiff (PAS) and Grocott-Gömöri Methenamine Silver (GMS) stains.
    5. Any two of:
      Aspergillus
      Fusarium
      Acremonium
      (1 mark each for any two)
      Feedback
      Broadly speaking, it is helpful to divide fungi that can cause keratitis into filamentous fungi and yeast-like organisms. Filamentous fungi can be further sub-divided into septate and nonseptate organisms. Septae are visible in this slide. These septae are non-pigmented, which suggests the organism is likely to be one of Aspergillus, Fusarium or Acremonium.
    6. Any three of:
      Contact lens wear
      Corneal abrasion or trauma
      Diabetes
      Dry eye
      Exposure to vegetable matter
      Immunosuppression (e.g. steroids)
      Previous corneal transplant
      Reduced corneal sensation
      Warm, humid environment
      (1 mark for each up to a maximum of three)
      Feedback
      Risk factors for fungal keratitis can be divided into host factors and environmental factors. Host factors include diabetes, dry eye (e.g. Sjögren's disease), immunosuppression (e.g. systemic or topical steroids), previous corneal transplant and reduced corneal sensation (e.g. previous herpetic disease; LASIK). Environmental factors include exposure to vegetable matter (e.g. farm workers) and warm, humid environments (e.g. travellers returning from the tropics).81
 
Optics
  1. Answer
    1. Right eye +3.00 DS/–1.50 DC axis 165°   Left eye −0.75 DS/–0.75 DC axis 5°
      (1 mark for each eye)
      Feedback
      To convert from positive to negative cylinder, or vice versa, simply add the given sphere and cylinder power together to give the new sphere, then change the sign on the cylinder and change the axis by 90°.
    2. +2.25
      Feedback
      To calculate the spherical equivalent, add half of the cylinder power to the sphere power.
    3. Right cataract surgery, iseikonic lenses, contact lenses, refractive surgery (1 mark for each, maximum 3 marks)
      Feedback
      Symptomatic anisometropia following cataract surgery is commonly addressed by removing the cataract in the fellow eye (if anisometropia is predicted then second eye surgery is often prioritised).
      Iseikonic lenses are a special lens form for glasses which can alter the image size to compensate for small degrees of anisometropia.
      Contact lenses reduce the impact of anisometropia as relative spectacle magnification varies according to back vertex distance, which is reduced in contact lenses compared to spectacles. However they are often poorly tolerated in the elderly.
      Various forms of refractive surgery could be used to reduce or eliminate the anisometropia, although in practice it is more likely that one of the first three options would be chosen.
    4. Index myopia
      Feedback
      Increasing nucleosclerosis can lead to an increase in the refractive index of the lens, resulting in increasing myopia. Conversely this means she will require less plus (or more minus) power in her correcting lenses, resulting in decreased anisometropia. The decision on whether any treatment at all is require at this stage will depend on whether she is symptomatic from anisometropia and/or from developing cataract in the right eye.82
    5. Corneal incisions placed at 75° and/or 165°, toric intraocular lens insertion (1 mark for each)
      Feedback
      Peripheral corneal incisions can be used to reduce the corneal curvature and therefore decrease the corneal refractive power on that axis. These can be placed as part of cataract surgery or in isolation as partial thickness incisions. Nomograms exist to predict the refractive impact depending on a number of factors, including incision length, depth and position, corneal pachymetry and patient age. Toric intraocular lenses require careful placement with respect to the angle of astigmatism, but they are unlikely to induce irregular astigmatism and can be placed through a normal phacoemulsification incision.
    6. Induced corneal astigmatism will increase with increasing distance between the limbus and the incision
      Feedback
      There is a risk of irregular astigmatism and glare if an incision is placed too near the centre of the optical zone.
  2. Answer
    1. (1 mark for each correctly labelled point)
      zoom view
      Feedback
      Gullstrand's schematic eye relies on describing the optical system of the eye in terms of cardinal points. Light originating from the first focal point will be refracted into parallel rays by the optical system, whereas parallel light entering the optical system will be focused at the second focal point (which lies on the retina in the schematic eye as it represents emmetropia). The first and second principal points lie close together in the anterior chamber, where the principal planes meet the principal axis. The first nodal point lies immediately anterior to the posterior pole of the lens, whereas the second nodal point lies in the vitreous just behind the posterior pole of the lens.83
    2. The nodal point in the reduced schematic eye lies in the posterior lens material
      Feedback
      The reduced eye uses a single principal point and a single nodal point to create a further simplified schematic representation of refraction in the eye. It essentially treats the eye as a single refracting surface with a power of +58.6 dioptres.
      zoom view
    3. The refractive power at an interface between two media is dependent on the difference in refractive index between the two media rather than the absolute refractive index. The cornea and tear film has an interface with the air, which has a refractive index of 1 compared to the cornea's refractive index of 1.37. The refractive index of the aqueous is 1.33, and the vitreous approximately 1.34, compared to the crystalline lens which varies from 1.38–1.42. Therefore there is a greater refractive effect at the air-cornea interface than at either the aqueous-lens or lens–vitreous interfaces (2 marks). Furthermore the radius of curvature of the cornea is smaller than that of the lens, and as the power of a refractive surface is inversely proportional to the radius of curvature this is also a factor in the greater contribution of the cornea to the total refractive power of the eye (1 mark).
  3. Answer
    1. The accommodative convergence/accommodation (AC/A) ratio (1 mark) is the number of prism dioptres of convergence induced by each dioptre of accommodation (1 mark)
      Feedback
      The AC/A ratio is generally a fixed relationship for each individual that stays constant from childhood to presbyopia.
    2. The normal range is 3:1 to 5:1
      Feedback
      This means that for each dioptre of accommodation, it is normal for this to be accompanied by 3 to 5 prism dioptres of accommodative convergence.
      The AC/A ratio can be temporarily altered by glasses or drugs that reduce the amount of accommodation needed, and permanently changed by strabismus surgery. Thus, there are optical, pharmacological and surgical management strategies for managing strabismus patients.84
    3. The AC/A ratio is 9.5:1
      (IPD, interpupillary distance; PCT, prism cover test).
      zoom view
      (IPD, interpupillary distance; PCT, prism cover test).
      Feedback
      An alternative way of expressing the heterophoria method formula is:
      AC/A = IPD + (near fixation distance in m) (near PCT – distance PCT)
      This uses 0.33 m, instead of 3 D, as the near fixation measurement.
      Remember that when expressing the size of the deviation (as PCT), this has a positive sign for esodeviations, and negative for exodeviations. For example, a 4 ∆ esophoria should be used as +4 in the AC/A calculation, whereas a 6 ∆ exotropia is expressed as −6.
      The gradient method is an alternative way of calculating the AC/A ratio. Whereas the heterophoria method uses the change in deviation for near versus distance, the gradient method takes measurements at the same distance, but induces changes in accommodation with different lenses. The IPD is not needed for the gradient method.
    4. This will dramatically reduce the esotropia (1 mark). This patient has a 30 ∆ esoptropia without glasses. As the AC/A ratio is 6, each dioptre of accommodation results in 6 ∆ of convergence. Prescribing the full hypermetropic glasses correction will reduce the amount of accommodation needed by 5 dioptres (1 mark), which will therefore reduce the amount of convergence by:
      zoom view
      In theory, therefore, this should eliminate the patient's esotropia.
      Feedback
      This child has a fully accommodative (refractive) esotropia. This occurs in significantly hypermetropic children and the deviation is by definition eliminated by glasses wear. Taking the glasses off will result in the squint recurring. The treatment is full-time glasses wear.
      If the esotropia is reduced but not eliminated by the hypermetropic correction, this is known as a partially accommodative esotropia and may require squint surgery in addition to glasses wear.85
  4. Answer
    1. Goldmann applanation tonometer
    2. At the tip of the tonometer head, two prisms are placed with their bases in opposite directions.
      This is to split the operator's view of the fluorescein-stained tear meniscus, which is formed by the contact between the tonometer head and the cornea (1 mark).
      This allows precise adjustment of the force on the dial, until the area of contact is seen to be correct (corresponding to 3.06 mm) by adjustment of the mires. It is only when the area of contact is correct that the force applied is proportional to the intraocular pressure (1 mark).
    3. Applanation tonometry is based on the Imbert–Fick principle:
      zoom view
      where P is the pressure, F is the force of application and A is the area of applanation.
      The force of application is directly proportional to the intraocular pressure (IOP) when the area of contact is exactly 3.06 mm. At this area of contact, the repulsive effect of corneal rigidity and attractive effect of the surface tension of the tear meniscus are balanced. The force is increased until the mires are aligned at their inner margins, indicating that area of corneal contact is correct. At this point, the reading on the dial is multiplied by ten to give the IOP in mmHg.
    4. 1
      Feedback
      In order to ensure the correct area of contact between the tonometer head and the cornea, the applanation force (force of application) must be adjusted until the mires are aligned at their inner margins.
      In part 1, the applanation force applied is too low, and the current reading will be lower than the true intraocular pressure (IOP). In part 2, the mires are aligned correctly and the IOP measurement can be taken. The configuration in part 3 is not seen due to the prisms splitting the view of the mires. In part 4, the applanation force applied is too high, and the current reading will be higher than the true IOP.
      zoom view
      86
    5. Any two of:
      Corneal thickness
      Excessive or inadequate fluorescein
      High astigmatism
      Pressure on the globe whilst holding the lids open
      Patient ‘squeezing’ or blepharospasm
      Breath holding (Valsalva)
      Failure to calibrate the tonometer
      (1/2 mark each)
      Feedback
      Care should be taken to avoid sources of error with Goldmann tonometry, as all of these factors may lead to under- or overestimation of the true intraocular pressure. Many of these are avoided with proper technique.
      For example, holding the lids open should be done with pressure on the bony orbit and not on the globe; adequate explanation and topical anaesthesia can help to maximise patient comfort and minimise ‘squeezing’ of the lids.
      In patients with a high degree of regular corneal astigmatism (e.g. >3 dioptres), the measurement should ideally be taken at 43° to the axis of the minus cylinder: if the prism is marked with degrees, line up the axis of the minus cylinder with the red line on the prism holder.
      Corneal pachymetry should be performed to help interpretation of the results.
    6. Due to the above-average corneal thickness, the measured intraocular pressure (IOP) is likely to be an over-estimate of the true IOP
      Feedback
      The accuracy of applanation tonometry relies on the central corneal thickness in the general population being within the normal range. The mean central corneal thickness is approximately 550 μm. This patient has significantly thicker than average corneas, meaning that the measured IOP is likely to overestimate the true IOP.
      Whilst it is essential to know the corneal thickness to aid interpretation of the IOP result, the use of specific correction algorithms or nomograms to then adjust the IOP result is controversial. What is not controversial is that corneal thickness should be taken into account in management decisions, with thin corneas being an independent risk factor for glaucomatous progression.